Preventive Med

You might also like

Download as doc, pdf, or txt
Download as doc, pdf, or txt
You are on page 1of 74

1 / 74

COMMUNITY AND FAMILY MEDICINE

HEALTH LAWS –

__D__ 1. It is the duty of the family to take care of its elderly members while the state may design programs of social
security for them, to motivate and encourage the senior citizens to contribute to nation building. This law is
called:
A. RA 8172 C. RA 7876
B. RA 8976 D. RA 7432

__A__ 2. This is an act creating the Philippine Institute of Traditional and Alternative Health Care (PITAHC) to accelerate
the development of traditional and alternative health care in the Philippines.
A. RA 8423 C. RA 7875
B. RA 8976 D. RA 7719

__A__ 3. Generics Act of 1988 is also called:


A. RA 6675 C. RA 7875
B. RA 7600 D. RA 8504

__B__ 4. This law provides basic health insurance for all Filipinos and establishes the PHIC:
A. RA 7600 C. EO 102
B. RA 7875 D. EO 105

__A__ 5. An Act promoting voluntary blood donation, provision for an adequate supply of safe blood and regulating blood
banks is:
A. RA 7719 C. RA 7160
B. RA 7875 D. RA 7277

__D__ 6. This law phases out leaded gasoline and reduces aromatics and benzene in unleaded gasoline, otherwise
known as Clean Air Act of 1999.
A. RA 5946 C. RA 7875
B. RA 7160 D. RA 8749

__C__ 7. RA 7600 is the:


A. Health Maintenance Organization Code of 2001
B. “Asin Bill”
C. Rooming-In and Breastfeeding Act of 1992
D. Phil AIDS Prevention and Control Act of 1998

__D__ 8. The National Health Insurance Program covers the following, EXCEPT:
A. Employed sector C. Non-paying members
B. Individually paying members D. Optional retirees

__D__ 9. The services that Phil Health does not compensate include/s:
A. Optometric services C. Nonprescription drugs
B. Normal delivery D. All of the above

__D__ 10. Under the NHI Program, benefit/s include/s:


A. Room and board for inpatient hospital care
B. Diagnostic and other medical examination services for in and out patient care
C. Emergency and transfer services
D. All of the above

RESEARCH METHODS –

__D__ 11. Criteria of a good research problem include/s:


A. Researchability C. Feasibility
B. Significance D. All of the above

__A__ 12. A researcher wrote this statement: “Coffee drinking does not cause coronary artery disease.” What type of
hypothesis is he stating?
A. Null hypothesis C. Two-tailed alternative hypothesis
B. One-tailed alternative hypothesis D. None of the above

__D__ 13. The research objective is essential for the following reason/s:
A. Indicates relevant variables to be considered in the study
B. Guide researcher in the choice of research design
C. Tell the researcher what data to collect and how to analyze them
D. All of the above

__C__ 14. The statement “To determine the effectiveness and safety of Vaccine A against Disease X.” is a:
A. Null hypothesis C. General objective
B. Alternative hypothesis D. Specific objective
Preventive Medicine
2 / 74

__A__ 15. In a cause-effect relationship study of cigarette smoking and lung cancer, cigarette smoking is the:
A. Independent variable C. Confounding variable
B. Dependent variable D. Control variable

__D__ 16. After being exposed to an infectious agent, the class was divided into 2 groups. Group 1 was given an
immunomodulator while the other group was given placebo. They were assessed after 1 week. What type of
research design was employed:
A. Cross sectional study C. Cohort study
B. Case Control Study D. Experimental Study

__A__ 17. A method of selecting a number of subjects from a universal population such that every member has an equal
chance of being drawn into the sample:
A. Simple random sampling C. Cluster sampling
B. Systematic sampling D. Multi-stage sampling

__C__ 18. A study on the hand washing practices of doctors assigned in the out patient clinic was anticipated. The most
appropriate data collection method to use is:
A. Review of documents C. Observation
B. Querry D. Laboratory examination

__D__ 19. Sources of inaccuracies when there is lack of objectivity of the test used:
A. Inter-observer variation C. Biological variation
B. Intra-observer variation D. A and B

__A__ 20. A profile study of victims of violence uses this tool for data analysis:
A. Frequency distribution C. Odd’s ratio
B. Chi-square test D. t test

BIOSTATISTICS –

__B__ 21. A Rural Health Physician wants to collect data on malnutrition among preschoolers in a particular Barangay.
The only weighing scale available is the bathroom scale. He tested the scale for repeatability or consistency
in weights it gives in three separate measurements. The Rural Health Physician is interested in what quality
of data:
A. Accuracy C. Validity
B. Precision D. Adequacy

__C__ 22. A physician wants to determine the prevalence of parasitism among school children in an urban area. There is
great variability in the economic status in that locality. On one hand, most children in private schools come
from affluent families, and o the other hand, most children from public schools come from poor families. The
most appropriate type of sampling methodology to use in this situation is:
A. Simple Random Sampling C. Stratified Random Sampling
B. Systematic Sampling D. Multi-stage Sampling

__C__ 23. In estimating/projecting population size, the method/s that assume/s that population size is changing
continuously at every infinitesimal amount of time is/are:
A. Arithmetic Method C. Exponential Method
B. Geometric Method D. Both B and C

__A__ 24. A measure of central tendency that is affected by outliers:


A. Means C. Mode
B. Median D. B and C

__C__ 25. A measure of variability, it is defined as the average squared deviation from the mean:
A. Range C. Variance
B. Standard Deviation D. Coefficient of Variation

__B__ 26. The distribution of childhood illnesses is an example of data that has this type of distribution:
A. Normal Distribution C. Skewed to the left
B. Skewed to the right D. None of the above

__D__ 27. The area between +/-3 standard deviations in a normally distributed data covers about this much percentage of
the total area:
A. Close to 90% C. Close to 98%
B. Close to 95% D. Close to 100%

__C__ 28. Suppose that systolic blood pressure (SBP) among non-hypertensive males is normally distributed with a
mean SBP of 110 mm Hg and a standard deviation of 10. About 68% of non-hypertensive males have SBP
within +/- 1 standard deviation. Thus 68% of non-hypertensive males have SBP that ranges from:
A. 90-130 mm Hg C. 100-120 mm Hg
B. 95-125 mm Hg D. 105-115 mm Hg

Preventive Medicine
3 / 74

__A__ 29. 20% (p=0.20) of men and 15% (p=0.15) of women are acrriers of a particular genetic trait. Suppose that this
trait can only be inherited by a child if both parents are carriers. What is the probability (p) that their child is
born with this genetic trait?
A. 0.03 C. Either 0.20 or 0.15
B. 0.35 D. None of the above

__A__ 30. A research is made to determine the relationship between emotional quotient (EQ) and academic performance
among medical students. After controlling for confounding variables, results of the study shows that the
relationship is statistically significant at p-value=0.06. The researcher thus concludes that there is a
relationship between the EQ and academic performance. A type of error that the researcher may commit in
this instance is:
A. Type I error C. Beta error
B. Type II error D. None of the above

PUBLIC HEALTH ADMINISTRATION –

__C__ 31. An evaluation or appraisal of present conditions and existing resources is:
A. Planning C. Situational analyses
B. Assessment D. Implementation

__D__ 32. Demographic factors in planning includes description of:


A. Existing health services
B. Working health systems
C. Socio-economic-environmental conditions
D. Age gender composition and distribution

__D__ 33. Health status evaluation includes the following, EXCEPT:


A. Infant mortality rate C. All of the above
B. Hospital services utilization D. None of the above

__B__ 34. A gap between what is and what should be:


A. Idea C. Priority
B. Problem D. Condition

__D__ 35. In the Primary Health Care approach, focus group discussion and community assembly are strategies for:
A. Appropriate technology C. Establishing support groups
B. Networking and linkages D. Community participation

__B__ 36. Working together with the people, learning their language and eventually establishing a herbal-medicinal
garden for people consumption is an example of:
A. Establishing support groups C. Networking and linkages
B. Appropriate technology D. Community participation

__C__ 37. An example of establishing support groups for program continuity:


A. Committing civic, religious, non-government organizations to actively participate
B. Putting up a Botika sa Barangay with the help of the Municipal council
C. Creation of an organization of asthmatic patients in the early identification and proper referral of
asthmatics
D. Consultative meetings with community leaders for the creation of latrines in certain areas

__B__ 38. Among the list, the program that would benefit the greatest number of people:
A. Provision of food and drugs
B. Accessibility to safe water
C. Health education
D. Proper waste collection and disposal

__D__ 39. In the course of implementation, when the quality of service is not at
par, management should:
A. Review established standards
B. Reprimand staff concerned
C. Measure difference in output
D. Modify activities/objectives

__A__ 40. During an evaluation procedure, complaints against nurses have been recorded and analyzed. In making
decisions, management should be reminded of:
A. Error of practical significance
B. Statistical data as evidence based materials
C. Opinions of co-managers and staff
D. Generalization based on records at hand

Preventive Medicine
4 / 74

BIOETHICS –

__A__ 41. A 75 year old diabetic patient with gangrenous foot refused to be amputated knowing all the risks and benefits.
She said she is too old to live without a leg. The doctor agreed. What is your ethical evaluation of the
doctor’s decision?
A. Right. Patient’s autonomy is respected.
B. Wrong. There’s harm to the patient.
C. Right. Patient accepted an additional burden.
D. Wrong. Patient’s ability to decide is questionable.

__C__ 42. A patient told his doctor that he do whatever is best for him. This is not a violation of the principle of autonomy
because of the following reason:
A. Accepts an additional burden C. Delegate authority
B. Gives up what is due D. Looses right to what is due

__D__ 43. The following are true EXCEPT:


A. Truth telling of harm overrides confidentiality
B. In emergency situation informed consent can be waived
C. Unplugging of life-sustaining machines is justifiable if and when they are no longer useful to the
dying person
D. Clinical research need not involve animal experimentation

__A__ 44. The principle of totality is the governing principle in the following situation:
A. Plastic surgery and mutilation with due cause
B. Voluntary organ donation
C. Confinement of mentally ill against their will
D. Conflict of interest

__B__ 45. Hospitals cannot be held liable for patients who go home against medical advice because patient:
A. Gave up what is due C. Accepted an additional burden
B. Lost right to what is due D. delegated authority

__C__ 46. Which among the following situations is a nonviolation of the principle of justice?
A. Kidneys for transplant are scarce. You decided to buy from indigent patients.
B. Allocating more of the government funds to kidney transplant program than to the free
immunization program for children under six.
C. Following a typhoon, health care is given to the victim who will die without an aid.
D. Compulsory tubal ligation in government hospital.

__B__ 47. Which among the following patients vying for one available respirator would gain the most from using the
respirator?
A. Trauma victim with severe brain damage. Only few brainstem functions remain. He is unlikely to
recover.
B. Patient with infection of the nervous system that rendered him paralyzed from the trunk down. He
is, however, progressively improving and is expected to recover.
C. Patient with complete and irreversible spinal injury that has left her paralyzed from the neck down.
She is able to talk but totally dependent on the respirator.
D. Cancer patient undergoing palliative treatment.

__C__ 48. A poor patient needing care agrees to enroll in a research study so as to be admitted in the hospital for free
treatment of his medical problem. Was an informed consent obtained in this case?
A. Yes. Information is understood by subject. No evidence of constraint on subject’s voluntariness.
B. No. Lack of competence.
C. No. Questionable voluntariness due to subject’s personal circumstance.
D. No. Probable incomplete disclosure and deception.

__B__ 49. An employee was discovered to be sick during the annual physical examination sponsored by the company.
His health would be adversely affected if he continues with his job. He pleads not to tell his employer.
What should the physician do?
A. Maintain confidentiality at all times.
B. Break confidentiality for the best interest of the patient.
C. Break confidentiality for the best interest of innocent third party.
D. Wait and observe.

__A__ 50. Which among the following is NOT a condition to stop treatment in a dying patient?
A. Life is preserved by ordinary means
B. Patient and/or family consents
C. Irrefutable evidence that biological death is imminent
D. Treatment will not prolong life for any significant time

Preventive Medicine
5 / 74

DOH Programs --

__B__51. One of these statements corresponds to the epidemiologic criteria of SARS diagnosis:
A. Severe respiratory illness
B. Close contact within 10 days of onset of symptoms with a person known or suspected to have
SARS
C. Radiographic evidence of pneumonia
D. Co V in specimens obtained more than 3 weeks after onset of illness

__A__52. The DOH responsibility on the National Rabies Control Program consists of:
A. Dog vaccination
B. Enforcement of ordinance on dog control measure
C. Management of dog bites
D. Officially designating canine rabies diagnostic laboratories

__B__53. This is the most effective control approach in Dengue Hemorrhagic Fever:
A. Adulticide C. Thermal fogging
B. Elimination of larval habitats D. Space spraying

__D__54. The leprosy situation in the Philippines according to demographic surveys showed:
A. 60% of new cases are MB
B. 8% of new cases are below 15 years old
C. 3% of new cases have grade 2 deformity
D. 10 0f the 16 regions achieved the elimination goal

__D__55. The elimination objective of Filariasis is:


A. Decrease prevalence rate in endemic municipalities to 2/1000 population
B. Decrease prevalence rate by 1/1000 population
C. Decrease prevalence rate by less than 1/1000 population
D. Zero case in previously endemic municipalities

__A__56. This is a new advocacy campaign of DOH in collaboration with heart specialty societies aimed at preventing
/controlling CVD:
A. Mag HL Tayo C. Yosi Kadiri
B. Love your heart, let it beat D. Walk for your life

__D__57. Reason/s for failure of TB Control efforts:


A. Inadequate political commitment and funding C. Inadequate organization of services
B. Over reliance on BCG D. All of the above

__A__58. Intervention methods in ARI:


A. Expanded Program on Immunization C. Good nutrition
B. Vitamin A supplementation D. Antibiotics intake

__A__59. The top leading types of cancer in men in the Philippines:


A. Lung & liver cancer C. Liver & prostate cancer
B. Lung & prostate cancer D. Liver & stomach cancer

__B__60. The most common lifestyle factor associated with lung cancer:
A. Alcohol drinking C. Frequenting night clubs
B. Cigarette smoking D. Drug abuse

__A__61. Breast Self Examination belongs to what level of prevention?


A. Primary C. Tertiary
B. Secondary D. A and B

__C__62. The drug of choice for Pneumonia as recommended by the WHO:


A. Benzathine Penicillin C. Cotrimoxazole
B. Amoxycillin D. Chloramphenicol

__C__63. The principal killer of HIV positive people:


A. Moniliasis C. Pulmonary Tuberculosis
B. Kaposi’s Sarcoma D. Pneumocystis carini

__B__64. A child having diarrhea with sunken eyes, thirsty and skin pinch goes back slowly is classified as:
A. Very severe dehydration C. Some dehydration
B. Severe dehydration D. No dehydration

__A__65. Long term goal/s of the AIDS control program:


A. Prevention of complications
B. Mandatory testing for AIDS
C. Wide use of condom
D. Disclosure of the results of HIV testing among visa applicants
Preventive Medicine
6 / 74

FAMILY MEDICINE --

__C__66. The De la Cruz couple, Pedro and Juanita have been married for two months. Which of the following would be
the main emotional issue at their stage of the family life cycle?
A. Balancing careers C. Committing to the marriage
B. Maintaining couple functioning D. Planning for a child

__D__67. After one year, the couple above have their first born, Louie. How would you classify their family?
A. Bilateral C. Extended
B. Blended D. Nuclear

__C__68. In constructing their genogram after 6 months have elapsed, a zigzag line is seen joining husband and wife.
This would denote which of the following?
A. Coalition C. Discord
B. Withdrawal D. Separation

__D__69. When Louie turned 2, he was confined for diarrhea. A genogram at this point would show him to be the index
patient. Which symbol is used?
A. Diagonal C. Broken line
B. Arbitrary D. Arrow

__A__70. If you wanted to measure this family’s functionality, which tool would be most useful?
A. APGAR c. SCREEM
B. Lifeline D. Ecomap

__C__71. If you wanted to check the available resources to respond to the illness of Louie, which tool is used?
A. APGAR C. SCREEM
B. FES D. GEI

__B__72. As Louie becomes a teenager, the family life cycle stage assumes a new developmental task, which is:
A. Developing autonomy C. Renegotiating couple bond
B. Expanding boundaries D. Starting mid-life transition

__B__73. In family systems theory, the coming of children to a married couple implies changes in the roles of both father
and mother. What do you call this kind of change?
A. First order C. Third order
C. Second order D. Pervasive

__C__74. This paradigm was developed by Engel who proposed that physiologic dysfunction is tied up with
environmental stress and internal issues.
A. Life-span perspective C. Biopsychosocial model
B. System approach D. Cross-cultural transaction

__D__75. Which of the following theories is useful for preventive care because it helps the clinician anticipate key
transitions and educate family members about changes?
A. Social support C. Psychoimmunology
B. Intergenerational D. Developmental

Epidemiology –

__D__76. Epidemiology can be defined as the study of:


A. The etiology of disease in humans
B. The frequency of causes of death in humans
C. The determinants of frequency of disease in humans
D. The distribution and determinants of frequency of disease in human populations

__B__ 77. The time interval between entry of an infectious agent into a host and the onset of symptoms is called:
A. The communicable period C. The preinfectious period
B. The incubation period D. The noncontagious period

__A__ 78. Primary prevention may be best undertaken during the period of:
A. Pre-pathogenesis C. Resolution or sequelae
B. Pathogenesis D. Any of the above

__D__ 79. In the study of the cause of a disease, the essential difference between an experimental study and an
observational study is that in the experimental investigation
A. The study is prospective
B. The study and control groups are of equal size
C. The study and control groups are selected on the basis of history of exposure to the suspected risk factor
Preventive Medicine
7 / 74

D. The investigators apply an intervention to influence the outcome of the study, for effective methods of
treatment, prevention, or clinical management

__B__ 80. The occurrence of a group of illnesses of similar nature at a rate above the expected number is called:
A. Hyperendemic C. Endemic
B. Epidemic D. Pandemic

__D__ 81. To determine whether maternal deficiency of folate is a cause of congenital defects of the neural tube, the
mothers of 100 newborns with congenital neural tube defects and 200 newborns without congenital neural
tube defects were questioned about intake of multivitamins and folate during pregnancy. What type of
study is this?
A. Clinical trial C. Cohort
B. Cross-sectional D. Case-Control

The following two-by-two table represents the findings of the study.

Mothers of Newborns Mothers of Newborns Total


with Congenital Neural without Congenital
Tube Defects Neural Tube Defects
(+) folate deficiency in the 15 85 100
mother
(-) folate deficiency in the 10 190 200
mother
Total 25 275 300

__A__ 82. The correct calculation of the parameter to measure the association between exposure and outcome is:
A. [15 x 190]/[85 x 10]
B. [15 + 85]/ [15 + 85 + 10 + 190]
C. [15 + 10]/ [15 + 85 + 10 + 190]
D. [15/(15 + 85)]/[10/(10 + 190)]

__A__ 83. The above-mentioned study revealed an odds ratio of 3.35 (95 % Confidence Interval= 1.35-8.42) associated
with maternal deficiency of folate. If the study described is accurate, which of the following statements is
true?
A. Results suggest that a baby whose mother had folate deficiency is about 3.35 times as likely to
be born with congenital defects of the neural tube as a baby whose mother did not have folate
deficiency and the association is significant (p<0.05).
B. Results suggest that the odds of giving birth to newborns with congenital defects of the neural
tube among mothers with folate deficiency is 3.35 as compared to mothers who do not have folate
deficiency and the association is significant (p<0.05).
C. Results suggest that the risk of giving birth to newborns with congenital defects of the neural tube among
mothers with folate deficiency is 3.35 times as compared to mothers who do not have folate deficiency and
the association is significant (p<0.05).
D. The results provide no evidence that maternal deficiency of folate is associated with congenital defects of
the neural tube in the offspring.

Lou Stewells, a pioneer in the study of diarrheal disease, has developed a new diagnostic test for cholera.
When his agent is added to the stools, the organisms develop a characteristic ring around them (He calls it the “Ring-
Around-the Cholera” [RAC] test). He performs the test on 100 patients known to have cholera and 100 patients known not
to have cholera with the following results:
Cholera No Cholera
(+) RAC test 91 12
(-) RAC test 9 88
Total 100 100

__A__ 84. The sensitivity of RAC test


A. 91/[91+9] x100%= 91%
B. 88/[9+88]x100%= 91%
C. 91/[91+12]x100%= 88%
D. 88/[12+88]x100%= 88%

During the investigation of an outbreak of food poisoning at a summer camp, food histories were obtained form
all campers as indicated in the table below.
Consumed Food Did Not Consume Food

Food served Ill Not Specific Attack Ill Not Specific


Ill Rate Ill Attack Rate
Hamburger 6 4 6/10 = 60% 4 4 4/8 = 50%
Potatoes 7 3 7/10 = 70% 2 3 2/5 = 40%

Preventive Medicine
8 / 74

Ice cream 8 10 8/18 = 44% 2 2 2/4 = 50%


Chicken 13 5 13/18 = 72% 1 7 1/8 = 12.5%
Lemonade 2 8 2/10 = 20% 4 4 4/8 = 50%

__D__ 85. The incriminated food item is most likely to be:


A. Hamburger
B. Potatoes
C. Ice cream
D. Chicken

MCU-FDT Medical Foundation


Department of Family & Community Medicine

1. Usually expressed as the percentage of persons diagnosed as having a specified disease who died as a result
of that illness within a given period:
A. Incidence rate C. Infection rate
B. Attack rate D. Case-fatality rate

2. Ability of the agent to invade and damage tissues of the host indicated by case fatality rate:
A. Pathogenic C. Infectivity
B. Virulence D. Immunity

3. A person not possessing sufficient resistance against a particular pathogenic agent to prevent contracting
infection when exposed to agent:
A. Suspect C. Susceptible
B. Reservoir D. Resistant

4. The ability of the agent to gain access and adapt to the human host to the extent of finding lodgment and
multiplying within the host:
A. Infectivity C. Virulence
B. Pathogenicity D. Infection

5. Rehabilitation of a stroke patient is what level of prevention:


A. Primary C. Tertiary
B. Secondary D. None of the above

6. Antibody of the newborns acquired firm the mothers:


A. Natural acquired immunity
B. Natural acquired passive immunity
C. Artificial acquired active immunity
D. Artificial acquired passive immunity

7. If all three (3) lines are filled up in the medical certification of death the entry that will be counted in the statistics
is:
A. Immediate cause C. Underlying antecedent cause
B. Intervening antecedent cause D. Other significant conditions

8. This study design is unable to link exposure with disease in particular individuals. Represents average
exposure levels.
A. Case-control C. Correlational
B. Case-series D. Cross-sectional

9. In a cohort study of breast implants and the risks of connective tissue, 48 y/o of the implanted patients are lost
to follow-up.
A. Selection bias C. Recall bias
B. Information bias D. Ecologic fallacy

10. In case-control study of mouthwash use and risk of oral cancers, the cases tend to over-report the use of
mouthwash:
A. Selection bias C. Recall bias
B. Information bias D. Ecologic fallacy

11. Approach used to compare the benefits of 2 alternative pharmacologic treatment for peptic ulcer disease:
A. Case-control C. Randomized controlled clinical trial
B. Cohort D. Descriptive study

12. The infant mortality rate of a country in a given year is 45.1 per year per 1000 live birth means:
A. 45.1 infants died before reaching their first birthday
B. 45.1% of all deaths occurred in infants below one year
C. 45.1 infants died before reaching their first birthday per 1000 live births
D. 45.1 infants per 1000 of the population died

Preventive Medicine
9 / 74

13. What is the proportion of unaffected individuals who will have the disease in two years when there are 1,050
subjects under observation and a total of 750 developed the disease?
A. 36% C. 71%
B. 100% D. 1.4%

14. Primary heath care is defined as:


A. An integral part both of the country’s health system of which it is the nucleus and of the
overall socio- and economic development of the community
B. The community health
C. An exhortation to engage in long term planning
D. Intensify health education efforts, particularly among the poorest segment of the population

15. The ongoing collection by government agencies of data relating to events such as births, deaths, marriages,
and health and health-0related conditions deemed reportable by local health authorities.
A. Descriptive statistics C. Vital statistics
B. Analytic statistics D. All

16. Measles vaccine is an example if a :


A. Killed vaccine C. Live, attenuated viral vaccine
B. Live vaccine D. Live attenuated bacterial vaccine

17. It is a system of pumps, pipes, devices and other structures for collection of liquid wastes:
A. Sewage C. Sewerage
B. Sewer D. Waste disposal

18. In the control of schistosomiasis, the drug of choice is:


A. Mebendazole C. Praziquantel
B. Oxantel pamoate D. Pyrantel pamoate

19. The strategy being implemented by the National Tuberculosis Program (NTP) to address poor treatment
compliance problem:
A. Provision of free-anti Koch’s medication
B. Systematization of drug supply distribution
C. Directly observed treatment
D. Health education to motivate patient

20. A child was brought to the E.R. after swallowing a metal detarnisher. The cause of poisoning is due to:
A. Solanine C. Lead
B. Cyanide D. Enterotoxin

21. The usual incidence of a disease in a population is referred to as the:


A. Pandemic rate C. Hypodermic rate
B. Epidemic rate D. Endemic rate

22. A company wants to put up a water source but the lot is situated near a cemetery. What is the distance
required from the cemetery to a water source?
A. 20 meters away C. 40 meters away
B. 30 meters away D. 50 meters away

23. The hallmark of the Control of Diarrheal Diseases Program is rehydration therapy. The components of ORS for
rehydration at home is:
A. 1 tsp. salt, 4 tsp., sugar, 1 liter pre-boiled water
B. 1 tsp. salt, 8 tsp., sugar, 1 liter pre-boiled water
C. 2 tsp. salt, 8 tsp., sugar, 1 liter pre-boiled water
D. 2 tsp. sugar, 8 tsp., salt, 1 liter pre-boiled water

24. A tourist was brought to the E.R. due to diarrhea after eating raw oyster. His poisoning is due to:
A. Trichinella C. Calcivirus
B. Shigellosis D. Clostridium botulinum

25. The best measure to characterize the prognosis of patients with congestive heat failure:
A. Incidence rate C. Survival rate
B. Case fatality D. Prevalence

26. To account for the placebo effect and to reduce the introduction of bias due to misleading conclusion when the
risk factor and disease are related at the population level but not within particular individuals. This is known as:
A. Single-blind study C. Double-blind study
B. Placebo-controlled trial D. Controlled-parallel trial

27. Ingestion of this solvent can result in permanent blindness due to optic atrophy:
A. Ethyl alcohol C. Ethylene alcohol
B. Isoprophyl alcohol D. Methyl alcohol

Preventive Medicine
10 / 74

28. This is the ability of the disease agent to give access and adapt to the human host thereby leading to its
lodgment and multiplication in the body:
A. Antigenicity C. Pathogenecity
B. Infectivity D. Virulence

29. The presence of a certain number of immune individuals in a community may after the spread of the disease.
This is referred as:
A. Vaccine C. Active Immunity
B. Attribute risk D. Herd Immunity

30. A student complains of headache and mental confusion after eating potato salad, poisoning is due to:
A. Ergotiem C. Gonyaulax
B. Solanine D. Muscarin

31. One of the general objectives of Primary Health Care is:


A. To train barangay health workers who will serve as interlinks between communities
B. To maintain health by promoting optimum standard of living and good health
C. To intensify health education efforts among the poorest segment of the population
D. To provide low cost but effective remedies for diarrheal diseases and respiratory infection

32. Case Control studies, participants are identified on the basis of :


A. Presence of the disease C. The amount of vaccine to be receive
B. Their exposure status D. The immunity

33. An experimental screening test for Cytomegalovirus has a sensitivity of 82% and a specificity of 93%. The
prevalence of CVM in the population to be screened is estimated to be 3%. The probability that an individual
with a positive test result has CVM is:
A. 26% C. 82%
B. 73% D. 18%

34. Which viral disease can be transmitted through the excreta?


A. Cholera C. Typhoid fever
B. Bacillary dysentery D. Hepatitis A

35. The serious chronic effect of ultraviolet radiation:


A. Conjunctivitis C. Inflammation of cornea
B. Erythema of the skin D. Skin cancer

36. This method eliminates almost all pathogens from the effluent:
A. Intermittent sand filtration C. Trickling filter
B. Contact beds D. Chlorination

37. The process of assigning treatment to patient with equal chance:


A. Selection C. Variation
B. Randomization D. Factorization

38. Disease associated with living organisms entering the body through ingestion of infected food:
A. Food poisoning C. Pasteurization
B. Sanitation D. Food borne disease

39. The Philippine TB Program uses this screening method for case finding:
A. Chest x-ray C. Sputum culture
B. Sputum microscopy D. Sputum gram staining

40. The hallmark of the Control of Diarrheal disease program is:


A. Stool examination C. Antibiotic treatment
B. Rehydration therapy D. IV Medications

41. The number of a new cases that develop in a population at risk for the disease over a specific period of time:
A. Point prevalence C. Incidence
B. Relevance D. Persistence

42. Incidence of a disease in a particular population over a specified time period:


A. Morbidity rate C. Attack rate
B. Mortality rate D. Crude rate

43. The cumulative incidence of the disease when the duration of a disease is short and the observation period
covers an entire epidemic:
A. Case fatality rate C. Mortality rate
B. Attack rate D. Morbidity rate

44. An individual who has contact with or who manifests the risk factor prior to becoming ill:
A. Exposed C. Infected
Preventive Medicine
11 / 74

B. Inflicted D. Well

45. Test used to assess the statistical significance of the difference between 2 population means in a study based
on data obtained from independent samples:
A. Chi-square C. T- test
B. Regression analysis D. Student T-test

46. Graph primarily intended to portray trend (i.e. change with time of variable):
A. Scatter plot C. Line graph
B. Histogram D. Frequently distribution

47. The following are elements of reproductive health EXCEPT:


A. Family Planning
B. Violence Against Women
C. Health Services Delivery Mechanisms
D. Education and Counseling on Sexuality and sexual Health

48. In a study about people’s attitude toward selling vaginal diaphragms in the supermarket, the first 50 adults who
entered the supermarket during the day of data collection were interviewed. The sampling design applicable in
this situation is:
A. Quota C. Convenience
B. Simple random D. Systematic

49. The usual incidence of a disease in a population is referred to as the:


A. Pandemic rate C. Hypodermic rate
B. Epidemic rate D. Endemic rate

50. The advantages of planning in public health is/are:


A. All of these
B. There is elimination of duplication of health problems
C. There is better distribution of resources
D. There is exclusion of unnecessary program

51. A 9 y/o boy consulted a Rural Health Unit because of cough associated with moderate to high grade fever. On
P.E. RR=50/min., flaring of the ala nasi, circumoral cyanosis, presence of supraclavicular and intercostals
retractions. What is the Airway Respiratory Infection (ARI) classification:
A. Severe C. Moderate
B. Mild D. Severe with complications

52. What is the reason for an inefficient health care delivery inspite of an adequate physician-population-ratio:
A. Maldistribution of physicians
B. Physicians migration to foreign countries
C. None of these
D. Inadequate number of hospitals

53. During what trimester of pregnancy should a vaginal bleeding be reported to a physician:
A. Any of these C. First
B. Second D. Third

54. Cocaine addiction among pregnant mothers are increasing lately. Infants born from these mothers are at risk to
develop:
A. Cardiac defects C. Mental deficiencies
B. Seizures D. Minor joint and umbilical abnormalities

55. Which among the following is a disadvantage of ground water:


A. Does not usually require regular treatment
B. Likely to be free from pathogenic organism
C. Continuous supply
D. It is likely to be hard

56. Which among the following techniques of disease prevention is highly effective against the spread of contact-
transmitted disease:
A. Closure of secondary transmission
B. Treat human sources of infection
C. Killing of the agent at the intimate sources
D. Increasing the health resistance

57. What unit of the health department received and analyze disease reports and submit or make results available
to doctors:
A. Consultants in communicable diseases
B. Public health staff
C. Epidemiology specialists
D. Disease intelligence services
Preventive Medicine
12 / 74

58. All of the following statements are TRUE regarding chlorination, EXCEPT:
A. It removes all pathogenic organisms from sewage effluent
B. Chlorine compound is the most common substance used
C. It does not eliminate odors
D. It inhibits the deposition of sewage effluent during tank treatment

59. At 100 dependency ratio of 105 can be interpreted as:


A. 100 persons are dependent on the 105 people
B. the computed value is not representative of the number of dependents to be supported
C. 105 persons will be supported by 100 able bodied persons
D. there are less number of dependents per 100 persons

60. The following are the important goals/purposes in the development of a public health surveillance system,
EXCEPT:
A. Detection of epidemics
B. Description of treads and the natural history of a health condition
C. Detection of rare but fatal conditions
D. Evaluation of control and preventive measures

61. The condition wherein there is a loss of normalcy or physiological, psychological or anatomical structural of
function is called:
A. Disability C. Handicap
B. Disease D. Impairment

62. Acute episodes of an asthma are closely associated with the following, EXCEPT:
A. Emotional stress C. Caffeinated sodas
B. Paracetamol D. Upper respiratory infection

63. The following are the basic health service offered, EXCEPT:
A. Social and educational welfare C. Health education
B. Environmental sanitation D. Maternal and child health

64. A Hepatitis B virus carrier is a person who is:


A. Hepatitis B core antigen positive for 6 months
B. Hepatitis B antigen positive for 6 months
C. Hepatitis B surface antigen positive for 6 months
D. None of those

65. The main goal of health for all of the Primary Health Care Program is:
A. Attainment of a level of health that will enable everyone to live on an economically productive
and socially satisfying existence
B. Enough medicine to be given free
C. Sufficient physicians, nurses, midwives in all municipalities
D. Nobody will get sick with infectious disease

66. What do you call the appearance/occurrence of a group of disease of the same nature where rate exceeds the
expected number:
A. Epidemic C. Hyperendemic
B. Endemic D. Pandemic

67. Adequate breast milk supply as maintained by:


A. Never elevating arms C. Using breast pumps
B. Breastfeeding regularly D. Drinking a lot of water

68. Biological filters are also called:


A. Rapid sand filter C. Mechanical filters
B. Slow sand filter D. Chemical filters

69. Prevalence is a measure of:


A. Cases occurring within an interval
B. Old and new cases in the community
C. Old cases only
D. New cases only

70. Nine families surveyed showed the number of children per family were 4, 3, 5, 3, 5, 6, 1, 3 and 7. The mean,
median and mode number of children per family are respectively:
A. 2, 3, 4 C. 4.1, 4, 3
B. 3, 4.1, 4 D. 4, 3.2, 2

71. In the calculation of the infant mortality rate, the denominator used is:
A. Mid-year population C. Total live births and still births in 1 year
B. Population between 0-1 year D. Total live births in one year
Preventive Medicine
13 / 74

72. The following are activities included in the under five clinic program, EXCEPT:
A. Medical care C. Growth monitoring
B. EPI D. Family planning

73. The following viral disease are transmitted by Aedes aegypti mosquitoes, EXCEPT:
A. Chinkungunya C. Yellow fever
B. Dengue D. Japanese encephalitis

74. Primary Health Care team in composed of:


A. Physicians, dentist and medical technologist
B. Health Center physician, nurse, midwife and volunteer health workers
C. Family Center physician and private medical practitioner
D. Provincial health officer, chief of district hospital and health center physician

75. Which among the following is measured by standard deviation:


A. Distribution C. None of these
B. Dispersion D. Central tendency

76. A characteristics of a statistical test denoting the probability that the null hypothesis will be rejected if it is indeed
false is:
A. Pearson correlation C. Spearman correlation
B. Power D. Regression analysis

77. When does a disease becomes a public health problem?


A. When the disease is spread and serious in effect
B. When the disease is spread and serious in the effect and it requires organized community
effort to minimize or control it
C. When the disease is very costly
D. When it requires organized community effort to minimize or control it

78. Epidemiology ca be defined as the study of:


A. The frequency f causes of death in humans
B. The pattern of organizations and financing of the health care
C. The distribution and determinants of frequency of disease in human
D. The etiology of disease in humans

79. What is the list of drugs that meets the health of the majority created by the Generic Law:
A. Essential drug C. Generic
B. Therapeutic drug D. Drug formulary

80-83. In preparation for a National Examination, 200 Medical Students completed 100 questions in a
practice test. Each student answered between 35 and 59 questions correctly. The arithmetic mean of correct
answers was 47 with standard deviation of 4. The number of correct answers per student was distributed normally.

80. The range of questions correctly answered is:


A. 12 C. 36
B. 24 D. 65

81. The percentage of students who correctly answered 39-55 questions is about:
A. 24% C. 95%
B. 68% D. 99%

82. The percentage of students who answered at least 51 questions is:


A. 1% C. 5%
B. 2% D. 16%

83. The percentage of student who answered less than 39 questions is:
A. 2.5% C. 16%
B. 5% D. 32%

84. A woman, pregnant for the first time is scheduled to have a Tetanus toxoid vaccine. She was given a complete
dose of DPT during childhood. The Tetanus toxoid that will be given to her now is considered as:
A. TT1 C. TT3
B. TT2 D. TT4

85. A child was given BCG vaccine. This will protect her from contracting:
A. Primary Koch’s Infection C. Miliary TB
B. TB Meningitis D. All of the above

86. An earmuff is considered as a:


A. Decoration C. Protective equipment
B. Fashion accessory D. Disturbance
Preventive Medicine
14 / 74

87. In order to protect children from contracting the mutant strain causing acute flaccid paralysis, the government
instituted the program “Patak Kontra.”
A. Diphtheria C. Polio
B. Pertussis D. Tetanus

88. In order to protect children from contracting the mutant strain causing acute flaccid paralysis, the government
instituted the program “Patak Kontra.”
C. Diphtheria C. Polio
D. Pertussis D. Tetanus

95. In a 3 X 4 contingency table, the number of degrees of freedom is:


A. 2 C. 6
B. 4 D. 8

96. A comparison of clinically diagnosed versus autopsy-confirmed gastric and peptic ulcers was performed on
10,000 consecutive deceased patients. 130 had a positive result and has the disease in question; 20 has no
ulcer on autopsy findings but was positive on clinical diagnosis; 170 were positive on autopsy but negative
clinically, and 9,680 had no disease both clinically and on autopsy. The prevalence of autopsy confirmed gastric
and peptic ulcer was closest to:
A. 3% C. 87%
B. 43% D. 98%

97. The interval between the time of contact and the entry of the agent and onset of illness is known as:
A. Incubation period C. Period of communicability
B. Latency period D. Period of immunity

98. A descriptive study is useful in:


A. Generating hypothesis C. Determining association
B. Testing hypothesis D. Testing intervention

99. This drug converts a sputum (+) patient to sputum (-):


A. INH C. PZA
B. Ethambutol D. Rifampicin

100. Potable water, sanitary toilet, vector and rodent control is included in:
A. Traditional Medicine C. Drug Abuse Control
B. Disaster Management D. Environmental Health

101. Communal faucet system or stand posts in a block of households are classified as:
A. Level I C. Level III
B. Level II D. Level IV

102. The skin slit and multi-drug therapy is used in the control and prevention of:
A. Dengue fever C. TB
B. Leprosy D. Malaria

103. The occurrence of a temporary increase in any disorder leading to ma significant change in the balance of
forces in a given population:
A. Endemic C. Sporadic
B. Epidemic D. Pandemic spread

104. A positive presumptive test for the presence of coliform group is the formation of:
A. Spore C. Sugar
B. Gas D. Cytochrome oxidase positive

105. A pediatrician computed the mean weight of his patients for the day and came up with 14.71 kg. This can be
interpreted as:
A. Fifty percent of patients weighed 14.71 kg. or less and 50% weighed more than 14.7 kg.
B. The usual weight of patients that day is 14.7 kg.
C. On the average the weight of the patients that day is 14.7 kg.
D. Sixty percent of patients weighed 14.71 kg. or less while 40% weighed more than 14.71 kg.

106. When distribution for the data is fairly symmetrical, the most appropriate measure of central tendency to be
used is:
A. Range C. Median
B. Mode D. Mean

107. In Philhealth, a single period of confinement is:


A. Confinement for different illness or injury
B. Confinement separated from each other by more than 90 days
C. Benefits can be availed only if the 45 days allowance is exhausted

Preventive Medicine
15 / 74

D. A series or successive confinements for the same illness not separated from each other by more
than 90 days

108. A computed sensitivity of 90% means that:


A. Ninety percent of persons without disease will be correctly labeled by the test as not diseased
B. Ninety percent of the persons with the disease will be detected by the test being used
C. Ten percent of persons who have the disease which will not be detected by the test
D. Ten percent of healthy people or people without the disease will be incorrectly labeled

CEBU INSTITUTE OF MEDICINE

BLUEPRINT FOR QUESTIONS, SY 2004 - 2005

PREVENTIVE MEDICINE

QUESTION NOS. BY TYPE OF QUESTION


SUBJECT MATTER NUMBER
0F
QUESTIONS

RECALL COMPREHENSION APPLICATION

Epidemiology 27 52 – 54, 60 - 64, 55 – 59, 65 – 66, 69 – 72,


67 – 68, 84 - 86 76 – 77, 83

Research 9 74, 79, 96 - 97 73, 78, 80 – 82

Vital Statistics (Rates & Ratios) 8 - - 87 - 94

Statistical Analysis 5 99, 100 75, 95, 98

MEAN MINIMUM PASS LEVEL (MPL) SCORE: 63.02 + 22.4

DEPARTMENT OF FAMILY & COMMUNITY MEDICINE


Cebu Institute of Medicine

Final Examination
PBL Level III

A. 1. This approach best utilizes health services through a system where individuals and families are referred to the
different levels of health care for professional attention:

A. Referral system
B. Syndromic approach
C. National health insurance
D. Minimum basic needs

D. 2. The most fundamental of the four basic principles of the primary health care approach:

A. Community and individual involvement and self-reliance


B. Intersectoral action for health
C. Appropriate technology and cost-effectiveness
D. niversal accessibility to available resources and services

A. 3. The family is the social context of health care because:

A. The family influences health behavior within the unit


B. The patients family is the greatest ally in the treatment
C. It is influenced by the level of functioning and family interaction
D. The family helps the patient decide on the issue of seeking medical support

Preventive Medicine
16 / 74

C. 4. Which assessment tool determines the capacity of the family to provide the necessary health care to its
members?

a. Family Draw a Circle Test


b. Family APGAR
c. SCREEM
d. Family Genogram

A. 5. Family members satisfaction with expression of intimacy and emotional interaction that exist is measured on the
Family APGAR by:

a. Affection
b. Adaptation
c. Resolve
d. Partnership

A. 6. This stage represents the most rewarding part of medical practice as the physician coordinates all aspects of
therapy:

a. Major therapeutic efforts


b. Reaction to diagnosis
c. Early adjustment to outcomes
d. Adjustment to the permanency of the outcome

D. 7. The responsibility of the physician in the Stage V of the illness trajectory is to:

a. Help the patient and his family deal with the immediate effects of trauma
b. Offer alternative interpretations of proposed treatment or intervention
c. Discuss explanatory models and fear of the patient
d. Aid the patient and the family in efficient and functional readjustment

C. 8. Disease in contrast to illness:

a. Includes the sufferers experience of the disease


b. Is deeply embedded in the socio-cultural context of the person who is ill
c. Is the primary biologic disorder
d. Can be explained by the patient’s explanatory model of his condition

B. 9. Which statement best describes the impact of chronic illness on the family?

a. There is little time for physical and psychosocial adjustment


b. The family suffers from a state of uncertainty about the condition of the patient
c. Often there is little support from within and outside the family
d. There is a need to deal with immediate decision

C. 10. The following are important medical transmitters of disease, EXCEPT:

a. Flies
b. Mosquitoes
c. Bedbugs
d. Fleas

D. 11. The following statements are true regarding TB infections, EXCEPT:

a. Human sputum is the most important source of infection


b. Patient with sputum positive on direct smears are much more infectious than those
positive only on culture
c. Children with primary complex are not infectious because they do not cough out the TB bacilli
d. Chemotherapy reduces infectiousness after two months of the short course chemotherapy

D. 12. One of the following statements is NOT TRUE with regards BCG immunization:

a. There is no absolute contraindication


b. It can be given to children in immune deficiency states as long as they are well nourished
Preventive Medicine
17 / 74

c. It can give 80% protection against tuberculosis


d. Its effects will last a lifetime

C. 13. The cheapest and the most reliable method for case finding for tuberculosis:

a. Chest X-ray
b. Tuberculin testing
c. Sputum microscopy
d. ELISA test

C. 14. Primary chemoprophylaxis in tuberculosis is:

a. Preventive treatment to individuals who are likely to develop tuberculosis


b. Also known as infection chemoprophylaxis
c. The giving of INH to individuals with contact with a contagious case of tuberculosis
d. INH given to high risk groups 8 weeks after a BCG vaccination

A. 15. The major mode of transmission of tuberculosis from person to person:

a. Airborne spread from droplet nuclei


b. Airborne transmission of TB bacilli on dust particles
c. Vehicle transmission through spoons and forks
d. Vectorborne through mosguito bites

D. 16. Under the Expanded Program of Immunization in our country, the fully immunized child should have received the
following vaccines:

a. One dose each of BCG, DPT & Polio and Measles Vaccine
b. Three doses of DPT and Polio
c. One dose of BCG, 3 doses of Polio and DPT, 3 doses of Hepatitis B
d. One dose of BCG, 3 doses of Polio & DPT, one dose of Measles Vaccine

C. 17. The most important single examination of water is:

a. Physical
b. Chemical
c. Bacteriologic
d. Biological

C. 18. The following substance even when present in drinking water may not adversely affect the health of the user:

a. Excess fluoride
b. Nitrates
c. Chlorine
d. Copper

C. 19. A more direct and sensitive measure of water pollution by sewage:

a. Poor ecologic status of marine life


b. Determination of median phosphate levels
c. Determination of the number of coliform organisms
d. Determination of biological oxygen demand

A children’s party was held in the afternoon in the classroom of your brother where they were served spaghetti, barbecue,
chicken salad, leche flan and ice cream in cups. After taking a very light dinner in the evening, your brother started complaining
of abdominal pain and started vomiting some bits of spaghetti and some other unrecognized food particles.

D. 20. As a sibling and a future doctor, which of the following data should be the least important?

a. Try to find out if there are other classmates taken ill or complaining of the same symptoms
b. The list of food your child has eaten
c. The list of food taken by those with the same complaints
d. Where the food was purchased

Preventive Medicine
18 / 74

D. 21. When staphylococcal food poisoning is suspected, which of the following is considered the least important in the
investigation?

a. History of gastrointestinal or respiratory ailment in the food handlers during the immediate period
preceding the food preparation
b. History of skin infection
c. Rectal swab in the food handler
d. Complete blood count in the food handler

C. 22. In the control of diseases:

a. Tradition based approach program has always been proven to be more cost effective in the long
run when dealing with communicable diseases.
b. It is only in non-communicable diseases that healthy lifestyle and behavior promotion is an
important consideration.
c. Monitoring of cases is doubly important in chronic communicable disease as
tuberculosis especially when considering compliance and response to treatment
d. The most important strategy for communicable disease control is environmental sanitation

C. 23. The following statements are true concerning diphtheria, EXCEPT:

a. Vaccination does not prevent patients from being carriers of the organism
b. Clinically overt diphtheria does not necessarily lead to immunity
c. Booster injection will be sufficient during preschool age to provide life long immunity
d. Antitoxin and vaccine are used for the therapy of the disease

B. 24. One of the following drugs is not included in the WHO recommended regimen for the treatment of leprosy:

a. Dapsone
b. Rifampicin
c. Clotazimine
d. Isoniazid

B. 25. The chief portal of exit of leprosy in the transmission of infection is:

a. The skin
b. Nasal mucosa
c. Gastrointestinal tract
d. Mouth

B. 26. Which of the following statements is NOT TRUE for rabies infection?

a. Only the dog has been identified as reservoir and vector in our country
b. The rabies virus can cross the placenta
c. Man to man transmission is possible when a healthy man develops rabies inoculation with the
saliva of a hydrophobic patient
d. Occasionally a virus aerosol may initiate the disease process.

D. 27. One of the following is not classified as a nutritionally relevant disease:

a. Measles
b. Tuberculosis
c. Diarrhea of infectious origin
d. Upper respiratory tract infection

D. 28. The severest lesion and highest mortality are seen in children below age 5 with this kind of deficiency state:

a. Riboflavin
b. Vitamin C
c. Iron
d. Vitamin A.

Preventive Medicine
19 / 74

C. 29. This is the one most affected with a deficiency of protein & calories:

a. Weight
b. Abdominal circumference
c. Muscle and fat
d. Height.

B. 30. The most important factor associated with perinatal morbidity and mortality:

a. Nutrition
b. Birthweight
c. Parity of the mother
d. Socio-economic status

A. 31. The most widely used anthropometric data to measure nutritional status in all age groups is the:

a. Weight
b. Weight for height
c. Skinfold thickness
d. Muscle mass.

A. 32. Which of the following statements is NOT true regarding tetanus toxoid immunization for pregnant women?

a. It can be given anytime during pregnancy


b. It should be given during the first two trimester of pregnancy
c. Two doses are given during the current pregnancy if she has not yet received any vaccination
d. One injection of tetanus toxoid should be given on the succeeding pregnancies after the first two
injections for a total of five injections during the childbearing years

B. 33. Pelvic inflammatory disease may be a sequelae of one of the following contraceptive methods:

a. Barrier methods
b. Intrauterine device
c. Oral contraceptive pills
d. Spermicidal jellies

C. 34. The oral contraceptive pill DOES NOT interact with which of the following drugs?

a. Rifampicin
b. Anticonvulsants
c. Beta lactam antibiotic
d. Barbiturates

A 38-year old woman G6 P6006, came to you for contraceptive advice. Her youngest child is two years old and she definitely does
not want another pregnancy. She smokes around 6 sticks of cigarette per day and is under medical treatment for
hyperthyroidism. She has a strong family history of cancer and her menstrual cycles are quite irregular. Her husband who works
in a nearby province is usually home on Friday evenings, but can occasionally be home on weekdays.

C. 35. Considering the lifestyle of your patient, the best method you could advise for your patient would be:

a. Oral contraceptive pill


b. Condom
c. Tubal ligation
d. Natural family planning

A. 36. If she would opt for an IUD, the best time for insertion would be:

a. During her menstruation


b. Anytime she goes to your clinic
c. At the middle of her menstrual cycle
d. Before the start of her menstruation

Preventive Medicine
20 / 74

C. 37. One of the following complications is not a consideration if your patient would choose an IUD:

a. Pelvic pain
b Infection
c. Venous thromboembolism
d. Uterine perforation

D. 38. One of the following is most affected by maternal conditions during pregnancy:

a. Infant mortality
b. Age specific death rate
c. Crude birth rate
d. Perinatal mortality

C. 39. The special vulnerability of the child is best represented numerically by:

a. Infant mortality rate


b. Infant morbidity statistic
c. Proportion of child mortality to general mortality
d. Perinatal mortality

C. 40. Of the following which is the least probable cause of neonatal mortality?

a. Difficult labor
b. Prematurity
c. Diarrheal diseases
d. Sepsis

D. 41. Which of the following condition is least likely to cause death in adolescents?

a. Malignancy
b. Accidents
c. Suicide
d. Infection

A. 42. After delivery, chemotherapeutic agents are applied to the eyes of the newborn to prevent:

a. Opthalmia neonatorum
b. Conjunctivitis
c. HIV infection
d. Neonatal syphilis

A. 43. Congenital defects in the infant may occur if the mother contracts rubella during this period of pregnancy:

a. The first trimester


b. Second trimester
c. Third trimester
d. There is no danger to the fetus

D. 44. The best method to monitor infant growth in order to facilitate the evaluation of the care rendered to the infant:

a. Complete immunizations
b. Birth weight
c. Weight for height monitoring
d. Monitoring of sequential values for weight during clinic visits

A. 45. This is the most common cause of mortality among school children:

a. Accidents
b. Congenital anomalies
c. Infection
d. Malnutrition

Preventive Medicine
21 / 74

D. 46. Perinatal mortality is best related to:

a. Conditions in the home environment


b. The type of maternal nutrition during the period of pregnancy
c. Socio-economic environment
d. Intrauterine development of the fetus

A. 47. Which of the following characteristics should best describe maternal and child health care facilities?

a. It must be adapted to the needs and the resources of the communities it serves
b. It should be patterned after programs of known success in other countries
c. Understanding of the attitudes and beliefs of the health care recipient should be a secondary
consideration
d. Morbidity and mortality patterns of hospital studies can be used as a reliable source of statistical
data for program planning of activities as they are representative of the area.

A. 48. This reflects causes of death related to maternal health prior to pregnancy as well as events during pregnancy,
delivery and the puerperium:

a. Perinatal mortality
b. Maternal mortality
c. Stillbirth
d. Fetal death

A. 49. The public health objective of the Expanded Program of Immunization is the:

a. Resulting protection of the community through herd immunity


b. Protection of the individual children from infection
c. Transmission of the antibody from the mother to infant
d. Protection of the mother from infectious disease

C. 50. Oral rehydration as a strategy of Primary Health Care is included under which program of the DOH?

a. Medical care
b. Essential drug
c. Garantisadong pambata
d. Health education

D. 51. In addition to immunization, this is considered the most crucial health promotion strategy both as an
interpersonal and a community-based effort:

a. Communicable disease control


b. Provision of safe water supply
c. Environmental sanitation
d. Health education

A. 52. Health promotion and specific protection are classified under:

a. Primary prevention
b. Secondary prevention
c. Tertiary prevention
d. Primordial prevention

C. 53. The level of prevention directed towards reducing or preventing residual defects arising from a disease is:

a. Primary prevention
b. Secondary prevention
c. Tertiary prevention
d. Primordial prevention

B. 54. The following are components of tertiary prevention, except:

a. Rehabilitation
b. Early diagnosis and treatment
c. Prevention of death
Preventive Medicine
22 / 74

d. Prevention of disability

A. 55. The reduction of air pollution through the strict implementation of government rules concerning the inclusion of
pollution control devices during the planning stage of the construction of factories is an example of this level of
prevention:

a. Primordial
b. Primary
c. Secondary
d. Tertiary

B. 56. The level of prevention which is directed towards preventing the progression of the disease is:

a. Primary prevention
b. Secondary prevention
c. Tertiary prevention
d. Primordial prevention

C. 57. This health promotion and disease prevention activity is most effective when organized as a community-based
effort:

a. Campaign against smoking in the prevention of lung cancer


b. Early detection of the disease
c. Immunizations
d. Identification of vectors of infectious diseases

A. 58. To prevent the initiation of a disease, measures must be applied during the:

a. Pre-pathogenesis period
b. Pre-symptomatic stage
c. Stage of clinical disease
d. Convalescent stage

A. 59. The following statements are true in the natural history of the disease, EXCEPT:

a. The period of pathogenesis mostly involves social and economic factors


b. The observation of the signs and symptoms constitutes the clinical horizon of the disease
c. Treatment interrupts the process, thus it is also preventive
d. With chronic diseases, the cause may be unknown

C. 60. Refers to the study of the determinants and distribution of diseases in human populations:

a. Biostatistics
b. Preventive Medicine
c. Epidemiology
d. Outcomes Research

A. 61. Refers to the ability of a diagnostic or screening test to identify correctly those who have the disease:

a. Sensitivity
b. Specificity
c. Positive Predictive Value
d. Negative Predictive Value

C. 62. Refers to the proportion of diseased individuals among all those who have positive test results:

a. Sensitivity
b. Specificity
c. Positive Predictive Value
d. Negative Predictive Value

D. 63. Refers to the proportion of normal individuals among all those who have negative test results:

a. Sensitivity
b. Specificity
c. Positive Predictive Value
d. Negative Predictive Value

B. 64. Refers to the ability of a diagnostic or screening test to identify correctly those who are truly normal or who do
not have the disease

Preventive Medicine
23 / 74

a. Sensitivity
b. Specificity
c. Positive Predictive Value
d. Negative Predictive Value

A. 65. If safe and effective treatment is available for a disease with fatal sequelae if left untreated, then the diagnostic
test for this disease should have a very high:

a. Sensitivity
b. Specificity
c. Positive Predictive Value
d. Negative Predictive Value

B. 66. If the only available efficacious treatment for the disease is associated with moderately severe adverse effects,
then the diagnostic test for this disease should have a very high:

a. Sensitivity
b. Specificity
c. Yield
d. Reliability

C. 67. Refers to the amount of previously unrecognized disease that is diagnosed and brought to treatment as a result
of the screening:

a. Sensitivity
b. Specificity
c. Yield
d. Reliability

B. 68. Refers to the active detection of disease or risk factors in asymptomatic apparently healthy individuals:

a. Active surveillance
b. Screening
c. Case finding
d. Passive surveillance

B. 69. Has strong ethical implications and therefore is not recommended if there is no treatment available for the
disease if detected:

a. Active surveillance
b. Screening
c. Case finding
d. Passive surveillance

B. 70. If this test characteristic of a particular diagnostic procedure is very high, then it is very likely that an individual
with a positive test result truly has the disease:

a. Sensitivity
b. Specificity
c. Positive Predictive Value
d. Negative Predictive Value

A. 71. If this test characteristic of a particular diagnostic procedure is very high, then it is highly probable that an
individual with a negative test result is truly disease-free:

a. Sensitivity
b. Specificity
c. Positive Predictive Value
d. Negative Predictive Value

D. 72. A common person-related determinant of disease which is not at all amenable to intervention or modification:

a. Occupation
b. Environmental exposures
c. Urban-rural location
d. Age

B. 73. This study design involves the comparison of the incidence rate of a particular disease between a group of
people who happen to have the exposure of interest at the start of the study and another group of people who do
not have the same exposure of interest at the start of the study:

a. Randomized controlled trial


b. Cohort
Preventive Medicine
24 / 74

c. Case-control
d. Cross-sectional

A. 74. The gold standard study design used to prove efficacy of a new drug when compared with standard therapy:

a. Randomized controlled trial


b. Cross-sectional
c. Cohort
d. Case-Control

D. 75. Determines the statistical power of a randomized controlled trial to detect a significant difference between two
drugs:

a. Level of significance
b. P value
c. 95% confidence interval
d. Sample size

B. 76. Measures the strength of association between an exposure factor and the disease or outcome in a cohort study
design:

a. 95% confidence interval


b. Relative Risk
c. Odds Risk
d. Level of significance

C. 77. Indicates the probability of exposure among the cases relative to the probability of exposure among the controls:

a. Number Needed to treat


b. Relative Risk
c. Odds Risk
d. Absolute Risk Reduction

B. 78. Ideal study design for comparing the prevalence of a particular disease between two population groups

a. Case series
b. Cross-sectional study
c. Cohort
d. Randomized controlled trial

B. 79. Comparative treatments are allocated or assigned to different groups of people based on a predetermined
selection process by the investigator:

a. Case-control
b. Randomized controlled trail
c. Cohort
d. Cross-sectional study

B. 80. A sampling procedure that requires determination of the k th interval:

a. Simple random sampling


b. Systematic sampling
c. Cluster sampling
d. Stratified sampling

D. 81. Sampling procedure that assures accurate representation of several different categories within the sampling
population

a. Simple random sampling


b. Systematic sampling
c. Cluster sampling
d. Stratified sampling

D. 82. The population to which the results obtained from a sample are generalized:

a. Sampling population
b. Study population
c. Elementary unit
d. Target population

A. 83. An example of a common-source epidemic:

a. Carbon monoxide toxicity within a basement parking area


Preventive Medicine
25 / 74

b. Measles
c. Hepatitis B infection
d. Severe Acute Respiratory Syndrome

C. 84. Refers to the separation of infected persons from those not infected for the period of communicability:

a. Quarantine
b. Surveillance
c. Isolation
d. Immunization

A. 85. Refers to the limitation of freedom of movement of apparently well persons or animals who have been exposed
to a case of infectious disease:

a. Quarantine
b. Surveillance
c. Isolation
d. Immunization

D. 86. Measures that reduce host susceptibility during an epidemic include:

a. Quarantine
b. Surveillance
c. Isolation
d. Immunization

Kindly refer to Table 1 (last page) for the answers to question nos. 87 - 94 questions:

B. 87. Crude Birth Rate (CBR) of Poblasyon Kaluluy-on:

a. 150/ 1000
b. 200/ 1000
c. 120/ 1000
d. 60/ 1000

A. 88. Crude Birth Rate (CBR) of Sitio Arang-Arang:

a. 150/ 1000
b. 200/ 1000
c. 120/ 1000
d. 60/ 1000

D. 89. Crude Birth Rate (CBR) of Mahayahay Estates:

a. 150/ 1000
b. 200/ 1000
c. 120/ 1000
d. 60/ 1000

A. 90. Community with the highest number of reported dengue cases:

a. Poblasyon Kaluluy-on
b. Sitio Arang-Arang
c. Ciudad Farang Sosyal
d. Mahayahay Estates

C. 91. Community with the highest dengue Case Fatality Rate (CFR):

a. Poblasyon Kaluluy-on
b. Sitio Arang-Arang
c. Ciudad Farang Sosyal
d. Mahayahay Estates

C. 92. Community with the highest Crude Death Rate (CDR)

a. Poblasyon Kaluluy-on
b. Sitio Arang-Arang
c. Ciudad Farang Sosyal
d. Mahayahay Estates

A. 93. Community that will require the most effective and most intensive maternal and child health care services:

a. Poblasyon Kaluluy-on
Preventive Medicine
26 / 74

b. Sitio Arang-Arang
c. Ciudad Farang Sosyal
d. Mahayahay Estates

B. 94. At most 30% of deaths in this community for the year 2003 were attributable to deaths in children less than 1
year of age:

a. Poblasyon Kaluluy-on
b. Sitio Arang-Arang
c. Ciudad Farang Sosyal
d. Mahayahay Estates

A. 95. Can be used to estimate the mean age of the target population:

a. Mean age derived from the study population


b. Mean age derived from the elementary unit
c. Mean age derived from the sampling frame
d. Mean age derived from the sampling population

C. 96. This part of any research study describes the intended usefulness and prospective users of the results of the
study:

a. Background
b. Conclusion
c. Rationale
d. Conceptual Framework

D. 97. The basis for data collection in any research activity:

a. Rationale
b. Study Maneuvers
c. Conceptual Framework
d. Research Objectives

C. 98. Describes the variability within a set of normally distributed values within a study population:

a. Coefficient of variation
b. Range
c. Standard deviation
d. Variance

C. 99. Parametric test used to determine for significant differences in proportions of categorical variables between more
than two (2) populations or data sets:

a. Z test
b. Student’s T Test
c. Chi-square (X2) Test of Homogeneity
d. ANOVA with Multiple Pairwise Comparisons

B. 100. Refers to the error of rejecting a true hypothesis in inferential testing:


a. Type II Error
b. Type I Error
c. β error
d. Bias

Table 1. Data from 4 Countries, 2004.

Poblasyon Sitio Ciudad Mahayahay Estates


Kaluluy-on Arang-Arang Farang Social

Midyear population 9000 4500 4000 3000

Total livebirths 1800 675 500 180

Total deaths 144 54 100 30

Total dengue cases 360 250 280 120

Total deaths due to dengue 36 15 56 6

Maternal deaths 16 5 4 1
Preventive Medicine
27 / 74

Deaths under 1 yr of age 86 16 10 2

Swaroops Index 10% 10% 15% 18%

LIST OF REFERENCES:

A. Medical Epidemiology, 3rd Edition: Greenberg, RS, Daniels. SR, Flanders, WD, Eley, JW, Boring, JR (eds),
Lange Medical Books/McGraw-Hill, Medical Publishing Division, 2001
B. Epidemiology: An Introductory Text, 2nd Edition: Mausner, JS, Kramer, S (eds), W. B. Saunders Company, 1985
C. Foundations of Statistical Analysis for the Health Sciences, Mendoza, OM, Borja, MP, Sevilla, TL, Ancheta, CA,
Saniel, OP, Sarol Jr., JN, Department of Epidemiology and Biostatistics, College of Public Health, University of
the Philippines, Manila, 2000
D. Clinical Epidemiology. A Basic Science for Clinical Medicine. 2 nd Edition. Sackett, DL, Haynes RB, Guyatt GH,
Tugwell P. (eds.), Little Brown and Company, 1991.
E. Primary Health Care Concepts and Challenges in the Changing World, Alma Ata Revisited. SHS Paper # 7
WHO
F. Kalusugan Para sa Masa. National Objectives for Health. Philippines, 1999-2004. Department of Health
G. Primary Health Care Reviews Guidelines and Methods, A. El Bindari-Hammad D. L. Smith, WHO, Geneva
H. Proceedings of the Orientation Course in Family Medicine, Philippine Academy of Family Physicians, Inc.
I. The Essentials of Contraceptive Technology, A Handbook for Clinic Staff, Population Information Program,
Center for Communication Programs The John Hopkins Bloomberg School of Public Health

DEPARTMENT OF FAMILY MEDICINE AND COMMUNITY HEALTH


Our Lady of Fatima University

Choose the BEST answer:

_D_1. Variations in socio-cultural prescriptions are influenced by the following factors:


A. Level of income D. All of the above
B. Occupation E. A and C only
C. Level of education
_E_2. Role of hospitals in Primary Health Care:
A. Provide curative services D. A and B only
B. Training of health workers E. A, B and C
C. Community people themselves
_C_3. The best people to identify and prioritize community health problems:
A. Community physicians D. Public health nurses and midwives
B. Social workers E. Medical and nursing students on rotation
C. Conduct researches
_B_4. It is the science and art of preventing diseases, prolongation of life, promotion of health and efficiency:
A. Public Health D. Family Medicine
B. Preventive Medicine E. None of the above
C. Community Medicine
_A_5. It is the science and art of preventing diseases, prolongation of life, promotion of health and
efficiency through organized community effort:
A. Public Health D. Family Medicine
B. Preventive Medicine E. None of the above
C. Community Medicine
_D_6. A continuing and comprehensive health care of the patient and his family delivered in a
personalized manner:
A. Public health D. Family Medicine
B. Preventive medicine E. None of the above
C. Community medicine
_B_7. A population growth rate of 1% means the doubling time is about:
A. 60 years D. 90 years
B. 70 years E. 100 years
C. 80 years
_C_8. If the Philippine population growth rate is 2.3, the doubling time is about:
A. 20 years D. 35 years
B. 25 years E. None of the above
C. 30 years
_C_9. Mainstay in the management of diarrhea:
A. Attapulgite D. Antibiotics
B. Kaolin pectin E. C and D only
C. Oral rehydration solution
_E_10. Factor/s to consider in problem prioritization:
A. number of people affected D. A and B only
B. killing power of the disease problem E. A, B and C
C. area coverage of the problem
Preventive Medicine
28 / 74

_B_11. This is the study of how society and people choose to employ scarce resources to
produce health care:
A. international health D. cost-benefit analysis
B. health economics E. health policy development
C. bioethics
_C_12. Ozone depletion due to urbanization and industry is most likely to lead to an increase in cases of:
A. coronary artery disease D. tuberculosis
B. diabetes mellitus E. none of the above
C. melanoma of the eyes
_E_13. Effective communicable disease control means:
A. marked reduction in indigenous transmission
B. disease incidence is no longer a public health problem
C. certain control measures can be modified or dropped
D. A an B only
E. A, B and C
_D_14. Medicinal plant product/s listed with the BFAD:
A. Lagundi D. A and B only
B. Yerba buena E. A, B and C
C. Ulasimang bato
_E_15. Presyong Tama, Gamot Pampamilya Program:
A. Parallel Drug Importation Project of the DOH and DTI
B. Tailored and targeted procurement of drugs by the government
C. These drugs are only available in seven DOH-retained hospitals
D. A and B only
E. A, B and C
_B_16. Generic names shall be written:
A. Enclosed in the parenthesis D. A and B only
B. Above the brand name E. A and C only
C. Below the brand name
_B_17. Type of drug dependence wherein body cells are already adapted to biochemical changes:
A. Chemical dependence D. Drug dependence
B. Physical dependence E. None of the above
C. Psychological dependence
_B_18. The component of public health administration which is concerned with the direction of the personnel and other techniques
related to operation:
A. Organization D. Coordination
B. Management E. Planning
C. Control
_A_19. “Process” as part of the systems model of a health care delivery system refers to:
A. Health activities D. Health indicators
B. Health resources E. All of the above
C. Health products
_B_20. The study of how society and people choose to employ scarce resources to produce care:
A. Cost-benefit analysis D. International health
B. Health economics E. Health planning
C. Health systems development
_B_21. Based on WHO guidelines, Vitamin A supplementation should be given to:
A. severe IDA D. leprosy cases
B. severe PEM E. TB cases
C. IDD
_B_22. Ferrous sulfate is better absorbed when it is taken:
A. with meals D. with coffee or tea
B. on an empty stomach E. with vitamin A
C. with milk
_B_23. Daily vitamin A intake of pregnant women should not exceed:
A. 5,000 IU/day D. 200,000 IU/day
B. 10, 000 IU/day E. None of the above
C. 100,000 IU/day
_A_24. The DOH Seal of Acceptance may be used by food manufacturers whose products contain:
A. essential nutrients, at least 1/3 of the RDA
B. vitamin A
C. iodine
D. iron
E. none of the above
_C_25. Which of the following is NOT considered as an excreta-transmitted disease:
A. Cholera D. Bacillary dysentery
B. Typhoid fever E. Paratyphoid fever
C. Hepatitis B
_D_26. Potential effects of UV light is/are:
A. early cataract formation
B. melanoma of the eyes
C. development of dysplastic nevi
D. A and B only
Preventive Medicine
29 / 74

E. A, B and C
_D_27. Primary level of prevention:
A. Health education D. A and B only
B. Immunization E. A, B and C
C. CBC, urinalysis, stool exam
_C_28.Demography is:
A. the geographical distribution, age and sex composition of a population
B. the state of an area with respect to the number of its inhabitants
C. an empirical, mathematical composition and statistical study of the size, composition and spatial distribution of
human populations and changes over time
_D_29. Variable/s that determine/s the population of any defined area:
A. fertility
B. mortality
C. migration
D. A, B and C
E. A and B only
_B_30. Major factor/s in the recent increase in world population:
A. migration
B. decline in death rates
C. fertility control
D. A, B and C
E. A and B only
_D_31. Demographic characteristic/s of the population:
A. Dependency ratio
B. Sex ratio
C. Population distribution by age and sex
D. A, B and C
E. A and C only
_A_32. Population pyramid is a graphical representation of:
A. age and sex composition
B. rural and urban population
C. migration and emigration status
D. A, B and C
E. A and C only
_C_33. A leading cause of morbidity in the Philippines which is not a leading cause of mortality:
A. diseases of the heart
B. Tuberculosis
C. Malaria
D. Accidents
E. Pneumonia
_A_34. True of the leading causes of morbidity and mortality in the Philippines:
A. Lifestyle diseases have overtaken infectious diseases as leading causes of mortality
B. Medical treatment of infectious diseases have improved
C. Diagnosis and management of lifestyle diseases are adequate
D. Preventive services are generally adequate
_B_35. Some leading causes of morbidity in the Philippines are not significant causes of mortality. What
could this mean?
A. Preventive services are adequate
B. Curative medical services are improving
C. Disease prognosis is not changing
D. Diagnosis is becoming accurate
_B_36. True of the leading causes of infant mortality in the Philippines:
A. Prenatal and obstetrical factors do not play a role in infant mortality
B. Environmental conditions still a major factor to infant mortality
C. Most causes of infant mortality are not preventable
D. Nutritional deficiencies do not cause mortality
_D_37. Not a cause of maternal mortality in the Philippines:
A. Hemorrhage
B. Abortion
C. Hypertension in pregnancy
D. Gestational diabetes
E. Complications of labor and delivery
_D_38. The normal distribution curve is determined by these parameters:
A. Mean
B. Standard deviation
C. Median
D. A and B only
E. A, B and C
_D_39. The total area under the normal distribution curve is:
A. 68% C. 99%
B. 95% D. 100%
_C_40. If the area bounded by 2 perpendicular lines and the x-axis were drawn 3X standard deviations
away from the mean ___% of the area is covered:
Preventive Medicine
30 / 74

A. 68% C. 99%
B. 95% D. 100%
_B_41. If we say that the confidence coefficient is 90% we mean:
A. We are sure that 90% of the data is correct
B. The data has 90% chance of occurring
C. 90% of the population will show the characteristics being observed
D. none of the above
_A_42. Advantage/s of Crude Rates:
A. Readily calculable for international comparisons
B. Useful in epidemiologic and public health services
C. Opposing rends in subgroups masked
D. A and B only
_B_43. Disadvantage/s of specific rates:
A. Difficult to interpret
B. Cumbersome to compare many subgroups of 2 or more population
C. Fictional rates
D. Absolute magnitudes dependent on standard population chosen
_D_44. Used for comparison of death occurrence:
A. Age-specific death rate
B. Sex-specific death rate
C. Age-sex specific death rate
D. Cause-specific death rate
_B_45. Proportionate Mortality indicator:
A. Age-sex specific death rate
B. Swaroop’s index
C. Infant Mortality Rate
D. Neonatal Mortality Rate
_E_46. From a Health Officer’s point of view, a low IMR will mean:
A. Adequate immunization program
B. Good environmental sanitation
C. Good disease control program
D. All of the above
E. A and C only
_D_47. From a social worker’s point of view, a low IMR will mean:
A. Illegitimates are not neglected
B. Female babies are not unwelcome
C. High standard of living
D. All of the above
E. A and B only
_D_48. Which of the following will make a country a priority for international aid?
A. IMR of 5/1000 livebirths
B. IMR of 15/1000 livebirths
C. IMR of 20/1000 livebirths
D. IMR of 50/1000 livebirths
_C_49. In Primary Health Care, the first contact of the community to the Health Care System is/
are:
A. Rural health physician
B. Community midwives
C. Barangay health volunteers
D. B and C only
E. A, B and C
CASE:
An organization of philantrophists decide to find a study of out-of-school youths in the National Capital Region. A 6-month
time frame is given to finish the study.

_B_50. The organization considers this a large-scale study in an extremely large population. The best
method to use should be:
A. Cluster sampling D. Simple random sampling
B. Multi-stage sampling E. None of the above
C. Stratified sampling
_A_51. The sample to be obtained should be representative of the population. This is guaranteed by:
A. Probability sampling C. Both
B. Non-probability sampling D. Neither
_C_52. The organization decided that males ages 16-18 years of age is the group to be studied. This
becomes the:
A. Sampling unit D. Sampling frame
B. Target population E. None of the above
C. Sampling population
_A_53. Since this covers a large population, they decided to cut down the sample to males 16years of
age. This is called:
A. sampling unit D. Sampling frame
B. Target population E. None of the above
C. Sampling unit
Preventive Medicine
31 / 74

CASE:
A study seeks to determine the effect of food and iron supplementation on the Hb levels of pregnant women in a rural
community.

_B_54. Which is the dependent variable?


A. food supplementation C. Iron supplementation
B. Hb levels D. Length of pregnancy
_A_55. How can we classify Hb levels as a variable?
A. quantitative continuous C. qualitative nominal
B. quantitative discrete D. qualitative ordinal
_C_56. If one of the variables were to be defined as with or without iron supplementation, what type of
variable would it be?
A. quantitative continuous C. qualitative nominal
B. quantitative discrete D. qualitative ordinal
_C_57. What would be the best method of data collection for this study?
A. use of documented sources C. observation and measurement
B. interview of subjects D. sending out questionnaires
_A_58. Which of the following is a descriptive statistic?
A. frequencies of the most common causes of mortality in the catchment area
B. correlation of smoking and lung cancer
C. comparison of the effects of two drugs
D. establishment of the cause and effect relationship between two factors

CASE:
A physician observed that in patients admitted at the medical center where he goes on duty there seems to be a
relationship between cirrhosis of the liver and hepatic encephalopathy. He would like to determine if there really is a relationship
and he wants to see immediate results.

_D_59. What study design will be most practical for his purpose?
A. case report D. case control
B. case series E. cohort study
C. cross-sectional studies
_C_60. The above design will be the best choice in this situation for the following reason/s:
A. The physician is studying a rare disease
B. The disease develops over a long time
C. The physician is investigating a preliminary hypothesis
D. None of the above
_B_61. The most convenient source of data for this study:
A. interview with patients D. examination of cases
B. review of case records E. all of the above
C. interview with co-workers
_A_62. The criteria for “cases” to be included in the study:
A. presenting signs, symptoms, laboratory must fit the definition of a “case”
B. must have the exposure being investigated
C. must be dead or on disability leave
D. A and B
E. A, B and C
_D_63. The control group may be:
A. Patients with encephalopathy but with no cirrhosis
B. Patients with encephalopathy and has cirrhosis
C. Patients with no encephalopathy but with cirrhosis
D. Patients with no encephalopathy

Data from preliminary investigation strongly suggests a relationship between encephalopathy and
cirrhosis. The physician now decided to make another study where he will start with subjects who have cirrhosis of the liver and will
follow them up to see the outcome.

_B_64. This study design is:


A. Case Control D. Case Series
B. Cohort E. Cross-sectional
C. Experimental
_D_65. The advantage of the above choice over other analytic studies:
A. It provides a strong evidence for possible cause and effect because it follows the
subjects forward in time
B. The temporal sequence between exposure and outcome can be more clearly
Established
C. It can be examine multiple risk factors of a single disease
D. A and B only
E. A, B and C

CASE:

Preventive Medicine
32 / 74

A physician was called to a seaside town to treat several college students for crampy abdominal pain and watery diarrhea.
The town officials were alarmed because they haven’t had any cases like this before. The physician was informed that there are 30
Biology students camped on the beach to gather marine specimens. The night before, they had a picnic where they had broiled
shrimps, crabs and raw oysters. Only 16 of the students were ill. On history taking, all 30 students ate shrimps and crabs but only
21 ate the raw oysters. After the 16 sick students were treated only 4 more cases were seen in the next 24 hours. All of those who
were sick ate raw oysters.

_B_66. If this were an outbreak/epidemic the main objective of the investigation is:
A. Check records and seasonal incidences
B. Identify ways of preventing further transmission of the disease causing agent
C. To set up a hypothesis
D. To establish a definite diagnosis
_C_67. In taking the history of these students, the most important aspect to ask is:
A. Travel D. Exposure to vectors
B. contracts with known cases E. all of the above
C. source of food and water
_C_68. The most probable source of the outbreak/epidemic is/are:
A. shrimp C. raw oysters
B. crabs D. A, B and C

CASE:

The total Philippine population as of the year 2002 is 80M. Approximately 15M females belong to the reproductive age
group with an average of 2.3M women getting pregnant annually. Common causes of infant morbidity and mortality are
pneumonia and diarrhea. With this in the scenario, answer the following:

_D_69. Considered as high risk pregnancy:


A. First pregnancy at age 17 D. A and B
B. Pregnancy with less than 2 years interval E. A, B and C
C. Third pregnancy
_A_70. Considered preventive measures for pneumonia:
A. Breast-feeding D. A and B
B. Vitamin C supplementation E. A and C
C. Herbal Medicine
_A_71. All pregnant women must receive:
A. Tetanus toxoid D. A and B
B. MMR vaccine E. A and C
C. Varicella vaccine
_E_72. Safe Motherhood Program endorses the following:
A. Birth-spacing should be at least 2 years
B. Tetanus toxoid must be given to all pregnant women
C. Early and regular pre-natal care is essential for a safe pregnancy
D. A and B E. A, B and C
_A_73. To lessen mortality from pneumonia, the CARI program involves:
A. Teaching simple but reliable diagnostic signs of pneumonia to mothers
B. Routinely giving antibiotics to all patients with cough
C. Vitamin C supplementation
D. A and B E. A, B and C

CASE:

The Local Chief Executive in your community assigned you to head the planning unit of the Health Department. Your task
is to give an orientation to the members of your unit, in preparation to the formulation of a five-year health development plan for your
community. Answer the following questions based on your knowledge on health planning:

_E_74. This is a characteristic of health planning that the planning unit should always take into
consideration:
A. Planning is time consuming D. A and B
B. Planning is a continuous process E. A, B and C
C. There is no single planning methodology
_E_75. A well – formulated objective is described as:
A. Specific D. B and C
B. Measurable and appropriate E. A, B and C
C. Realistic and time-bounded
_A_76. Community Diagnosis is usually regarded to as:
A. Situational analysis D. B and C
B. Focused group discussion E. A, B and C
C. Health survey

CASE:

Preventive Medicine
33 / 74

A representative of the DENR visited a chemical factory in Laguna. Upon inspection he found out that there
was a leakage of chemicals into the drainage system which empties directly into a nearby river. Said river is being utilized
by barrio folks for bathing and washing clothes. A sample of the water taken from the river was analyzed in a laboratory.

_B_77. Water sample was found to be cloudy and salty. Salty taste of water is due to presence of:
A. Iron D. A and B
B. Chloride E. B and C
C. Calcium
_C_78. Single most important test to find out if the water is potentially dangerous:
A. Physical examination D. Radiological examination
B. Biological examination E. Chemical examination
C. Bacteriological examination
_B_79. Water from the river consumes a lot of soap when used in washing clothes. This property of water
is referred to as:
A. Turbidity D. Alkalinity
B. Hardness E. Salinity
C. pH
_A_80. An unpleasant odor was emitted by the water within the factory. This may be due to the presence
of:
A. Algae D. Amoeba
B. Lead E. Chlorine
C. Iron

CASE:

The DENR representative also found out that toxic fumes produced by combustion of chemicals from the same factory are
emitted at night. Evidence is seen in the nearby trees wherein the leaves are crumpled and do not bear fruit.

_E_81. Effect of acid rain on lakes and aquatic ecosystems:


A. Forms mucus in fish gills D. A and B
B. Causes decreased calcium levels in water E. A, B and C
C. Causes sudden drastic change in pH of the lake
_B_82. Fish can not reproduce at this pH level:
A. <6 D. >8
B. <5.5 E. None of the above
C. 7
_A_83. Accumulation of this metal in organs and tissues of humans has been linked to brain damage:
A. Mercury D. A and B
B. Aluminum E. B and C
C. Lead

CASE:

You are a medical practitioner in a remote town in Basilan. Its population is composed mostly of children less
than 5 years old. 50% of the women belong to the reproductive age.

_D_84. To bring down the maternal mortality rate, you would like to inform the mothers about the danger
signs of pregnancy, such as:
A. Blurred vision and dizziness D. A and B
B. Puffiness of the face and hands E. a, B and C
C. Dysuria
_C_85. Pregnancy- related health risks, EXCEPT:
A. Pregnancy with less than 2 years interval C. Pregnancy at age 23 years
B. 4 or more pregnancies D. Pregnancy of a patient with renal disease
_B_86. Since majority of the women are below 20 years old, you would advice them against getting
pregnant because:
A. Generally, this age group have more difficult pregnancies and deliveries
B. They may not be psychologically prepared to become mothers
C. They take longer to recover following childbirth
D. A and C E. B and C
_D_87. Based on your knowledge of the CDD program, treatment of diarrhea involves:
A. Routine use of anti-diarrheal drugs
B. Administration of oresol
C. Reserving the use of antibiotics for cases of dysentery and cholera
D. B and C E. A, B and C
_E_88. Based on your knowledge of the CARI protocol, death from pneumonia is preventable through:
A. Early recognition of cases D. A and C
B. Prompt treatment of non-severe cases at home E. A, B and C
C. Quick referral of severe cases
_C_89. The earliest and a reliable predictor of pneumonia based on the CARI protocol:
A. Cyanosis D. A and B
B. Chest in-drawing E. A, B and C
C. Tachypnea
Preventive Medicine
34 / 74

_C_90. You would advise mothers to continue breast-feeding until what age?
A. 4 to 6 months D. 30 months
B. 12 months E. None of the above
C. 24 months

CASE:

As a young doctor in DOH, you are assigned to Sitio Batibot, a 5-hectare land bounded on the east by a creek, on the
north by Jorenz St., on the south by a paint and rubber factory and on the west by Lakandula St. It is occupied by people from
different ethnic groups: Visayans, Ilokanos, a few from Bicol and Mindanao. There are 200 families with an average family size of 6
to 7 members, with a total population of 5,000. Sixty percent of the heads of family are unemployed, 40% are employed as plumber,
mason, carpenter and tricycle drivers. Majority finished elementary with a few reaching the collegiate level. Environmental sanitation
is a big problem, piles of garbage are scattered everywhere, canals are stagnant, the creek is overflowing with garbage and excreta,
rodents and insects around, some household still have no electricity, water supply comes mainly from a deep well with a few houses
supplied by the MWSS. There are 6 communal toilets, mostly pour flush type. Majority still resort to “balot”system and those living
near the creek use overhung latrine. There are practising “herbolario”, 2 “hilots”, and 1 midwife residing within the community. The
nearest Health Center is 10 minutes ride and the school is 15 minutes walk away from the community.

_A_91. You were tasked to promote Primary Health Care in Sitio Batibot. The first thing that you will do
in the promotion of Primary Health Care is:
A. General awareness campaign on Primary Health Care in the community.
B. Organize the community of Sitio Batibot
C. Do a Situational Analysis or Community Diagnosis
D D. Let the community identify their own health problems
E E. Let the community prioritize their own problem
_E_92. Which of the following is NOT an objective of Primary Health Care?
A. Promote/maintain health especially in remote communities
B. Develop community leadership and initiative
C. Develop communities to be self-reliant
D. Provide health services to complement community effort
E. Create dependence on foreign aids and loans
_E_93. Which of the following is/are considered as characteristic/s of Primary Health Care?
A. Community-based D. Emphasizes self-reliance and self-determination
B. Acceptable and Affordable E. All of the above
C. Accessible
_E_94. Which of the following can be considered as essential element/s of Primary Health Care?
A. Family planning program D. A and C
B. Provision and proper use of herbal medicine E. A, B and C
C. Promotion of adequate food supply
_A_95. The first contact of the community and the initial link to the health chain is the:
A. Barangay Health Worker D. Nurse
B. Clinical clerks/4th year medical students E. Rural Health Physician
C. Midwife
_D_96. Intrasectoral cooperation and linkage in Sitio Batibot will mean:
A. Cooperation between government organizations and the people’s organization
B. Cooperation between non-government organizations and the people’s organization
C. Cooperation between government and non-government organizations
D. Cooperation between government organizations, non-government organizations and people’s organization
E. Cooperation within the people’s organization themselves

CASE:

After graduation, you decided to go back to your home province to start your private practice. You notice that there are
many children with signs of micro and macro-nutrient malnutrition.

_A_97. A 3-year old child had recurrent respiratory tract infections over the past 6 months. His weight is
only 50% of the ideal and exhibits marked muscle wasting and lethargy. Laboratory exams
revealed serum albumin of 3.8 g/dL and a hemoglobin of 9.2 g/dL. These findings are most
suggestive of:
A. Marasmus D. Kwashiorkor
B. Vitamin A deficiency E. Anorexia nervosa
C. Folate deficiency
_E_98. A 4-year old child is only 70% of normal body weight. On PE, you noticed that he has dependent
edema of the lower extremities as well as an enlarged abdomen. The flaking skin shows irregular
areas of depigmentation, hyperpigmentation, and desquamation. These findings are most
suggestive of:
A. Marasmus D. Cretinism
B. Anemia E. Kwashiorkor
C. Vitamin Atoxicity
_C_99. Which of the following contains antioxidants?
A. Lard D. Bread
B. Ice cream E. Meat
Preventive Medicine
35 / 74

C. Carrot juice
_C_100. Iodine deficiency disorder during pregnancy can cause:
A. Anemia D. Blindness
B. Xerosis of the skin E. Muscle atrophy
C. Stillbirth

Subject: PREVENTIVE MEDICINE AND COMMUNITY HEALTH


Instructions : CHOOSE THE BEST ANSWER

The purpose is to limit the incidence of disease by controlling causes and risk factors
Primordial prevention
Primary prevention
Secondary prevention
Tertiary prevention
Answer: B

The property of a test to identify the proportion of truly ill persons in a population who are identified as ill by a screening
test
Sensitivity
Specificity
Positive predictive value
Negative predictive value
Answer : A

The probability of a persons having the disease when the test is positive
Sensitivity
Specificity
Positive predictive value
Negative predictive value
Answer: C

The extent to which a test is measuring what it is intended to measure


Reliability
Validity
Sensitivity
Specificity
Answer: B

A study that measures the number of persons with influenza in a calendar year
Cohort study
Case control
Cross sectional
Case report
Answer: C

Stage by which the presence of factors favor the occurrence of disease


Stage of susceptibility
Stage of presymptomatic disease
Stage of clinical disease
Stage of disability
Answer: A

Modes of horizontal transmission of disease, except


Contact
Vector
Common Vehicle
Genetic
Answer:D

An infected person is less likely to encounter a susceptible person when a large proportion of the members of the group
are immune
Active immunity
Passive immunity
Herd immunity
Specific immunity
Answer: C

Occurrence in the community of a number of cases of disease that is unusually large or unexpected
Endemic
Epidemic
Pandemic
Infection
Preventive Medicine
36 / 74

Answer: B

Measures of central tendency, except


Mean
Median
Mode
Variance
Answer: D

Range of values surrounding the estimate which has a specified probability of including the true population values
Standard deviation
Standard error
Confidence interval
Correlation coefficient
Answer: C

The probability of rejecting the null hypothesis when it is true


Type 1 error
Type 2 error
Power of a statistical test
Level of significance
Answer: A

The following are measures of disease frequency, except


Incidence rate
Prevalence
Cumulative incidence
Relative risk
Answer: D

The proportion of cases of a specified disease or condition which are fatal within a specified time
Morbidity rate
Case fatality rate
Proportionate mortality
Death rate
Answer: B

The relation between exposure and disease is considered to be causal or etiological in the following, except
Dose response relation
Cessation of exposure
Temporal relation
No confounding
Answer: D

A study that measures the incidence of a disease


Case report
Cross sectional
Case control
Cohort
Answer:D

A study wherein bias is less likely to occur


Case report
Cross sectional
Case control
Cohort
Answer: D

The proportion of disease incidence that can be attributed to a specific exposure


Relative risk
Odds ratio
Attributable risk
Potential risk
Answer: C

All of the following are potential benefits of a randomized clinical trial, except
The likelihood that the study groups will be comparable is increased
Self-selection for a particular treatment is eliminated
External validity of the study is increased
Assignment of the next subject cannot be predicted
Answer: C

Preventive Medicine
37 / 74

Recall is an example of what type of bias


Selection bias
Information bias
Confounding
Systematic
Answer: B

Type of design where both exposure and disease are determined simultaneously for each subject
Case study
Cross sectional study
Case control study
Cohort study
Answer: B

A study is conducted to determine the proportion of persons in the population with PTB using AFB sputum for diagnosis
Case study
Cross sectional study
Case control study
Cohort study
Answer: B

Randomization is the best approach in designing a clinical trial in order to


Achieve predictability
Achieve unpredictability
Achieve blinding
Limit confounding
Answer: B

Type of sampling whereby subjects are assigned according to a factor that would influence the outcome of a study
Simple random sampling
Systematic sampling
Stratified random sampling
Cluster sampling
Answer: C

The extent to which a specific health care treatment, service, procedure, program, or other intervention produces a
beneficial result under ideal controlled conditions is its
Effectiveness
Efficacy
Efficiency
Effect modification
Answer: B

What is the desired body weight of a 7 month old infant weighing 3600 gms at birth?
8.8 kgs
9 kgs
6.5 kgs
7.8 ks
Answer: D

Infants double their weight at


6-7 mos
9-10 mos
5-6 mos
3-4 mos
Answer: C

What is the total energy requirement for a 50 kg housewife without househelp?


2000 cal/day
1800 cal/day
2100 cal/day
2200 cal/day
Answer: C

The symptom that appears to consistently differentiate between PTB and non-TB respiratory disease
Night sweats
Anorexia
Chronic cough
Hemoptysis
Answer: C

Preventive Medicine
38 / 74

Asymptomatic PTB is most prevalent in what group?


Infants
Smokers
Young Children
Older age group
Answer: D

What should be the first test done when confronted with a patient in whom there is clinical suspicion of PTB?
Chest X-ray
Sputum AFB
Sputum GS/CS
All of the above
Answer:

What is the ideal timing of the three sputum samples?


First specimen after time of consultation at home, Second specimen early morning sputum collected by the
patient and Third spot specimen upon submission of second sputum
First specimen at home, Second specimen on the spot at time of consultation and Third spot specimen upon
submission of second sputum.
First specimen on the spot at time of consultation, Second specimen early morning sputum collected by the
patient and Third spot specimen upon submission of second sputum.
First specimen on the spot at time of consultation, Second specimen two days after consultation collected by
the patient at home and Third spot specimen upon submission of second sputum.
Answer: C

A Sputum AFB result of (++) means


3-9 bacilli in entire smear
1-9 bacilli /OIF
1-2 bacilli in entire smear
1-9 bacilli /10 OIF
Answer: D

What is a significant minimum microscopy result for the presumptive diagnosis of PTB?
A. A report of AFB ++ or more for any one of the submitted sputum specimens
B. If at least one of the 3 specimens is positive
C. A report of AFB +++ or more for any one of the submitted sputum specimens
D. A report of AFB + or more for any one of the submitted sputum specimens
Answer: A

TB cultures should be done in any of the following situations except


Smear (+) patients with fall and rise phenomenon
All cases of previously treated for >3months but <6months
Smear (+) patients with symptoms highly suggestive of PTB and suggestive C-Xrays
All cases of relapse
Answer: C

A patient with positive PTB exposure and PPD but lacking in signs of active disease and target organ damage is classified
by the American Thoracic Society as
A. PTB I
B. PTB II
C. PTB II
D. PTB IV
Answer: B

37. In the prevention of Cardiovascular disease, salt restriction should be less than how many grams per day?
A. 2 gms
B. 3gms
C. 4gms
D. 5gms
Answer: D

38. In Cardiovascular disease prevention, one should progressively increase moderate physical activity to
A. 30mins once a week
B. 30 mins twice a week
C. 30 mins three times a week
D. 30 mins daily
Answer: D

39. In patients with history of TIA, heart attack or diabetes, the goal in Blood pressure is
A. < 140 and <90
B. <130 and <80
C. <120 and <70
Preventive Medicine
39 / 74

D. < 110 and < 60


Answer: B

40. Leading cause of Diarrheal disease


A. Enterotoxigenic Escherichia coli
B.Salmonella (non-typhoid)
C.Rotavirus
D. Campylobacter jejuni

Answer: C

41. What degree of dehydration is often seen in a child exhibiting more thirst and sunken eyeballs?
A. No signs
B. Some signs of dehydration
C. Severe signs of dehydration
D. Equivocal
Answer: B

42. In Treatment Plan A, the amount of ORS to be given to a child less than 2 y.o. after each loose stool is
A. 100-200ml
B. 50-100ml
C. 300ml
D. 250ml
Answer: B

43. If the child vomits, you may continue slowly giving small amounts of ORS solution after a minimum of
A. 1 hr
B. 30 mins
C. 2 hrs
D. 10 mins
Answer: D

44. How many grams Sodium Chloride is in the standard World Health Organization ORS formula?
A. 5
B. 20
C. 1.9
D. 3.5
Answer: D

45. Which of the following is true regarding appropriate Nutritional Therapy during Diarrhea?
A. decreases stool output
B. shortens duration of illness
C. allows significant weight gain
D. All of the above

Preventive Medicine
Answer: D

46. Judge Reyes wants for all his children and their respective families to live with him and his wife
until the time of his death. This is an example of what structure of family?
A. Nuclear
B. Extended
C. Single Parent
D. Blended
Answer: B
47.Juan and Maria with their five year old son live in their own home and are managing to survive with Juan’s daily wage. This is an
example of what structure of family
A. Nuclear
B. Extended
C. Single Parent
D. Blended
Answer: A

48. Start of the family life cycle


A. Newly Married Couple
B. Family With Young Children
C. Unattached Young Adult
D. Family With Adolescents
Answer: C

49. Mammography should be done annually in women of what age?


A. 50 y.o. and above
B. 60 y.o. and above
C. 45 y.o. and above
D. 30 y.o. and above
Answer: A

50. Women with prior pregnancy affected by neural tube defects who are planning pregnancy should take this chemo-prophylactic
A. Vit. C
B. Retinoin
C. Vit. B complex
D. Folic Acid
E. Ferrous Sulfate
Answer: D

51. An act penalizing the refusal of hospitals and medical clinics to administer appropriate initial medical treatment and support in
emergency or serious cases, amending for the purpose batas pambansa bilang 702, otherwise known as "an act prohibiting the demand of
deposits or advance payments for the confinement or treatment of patients in hospitals and medical clinics in certain cases". Is also known
as:
A Ra 8347
B. Ra 3742
C. Ra 8344
D. Ra 7844
Answer: C

52. An act to advance corneal transplantation in the Philippines, amending for the purpose republic act numbered seven thousand one
hundred and seventy (R.A. no. 7170), otherwise known as the organ donation act . This was enacted on:
A. January 14, 1996
B. December 15, 1997
C. August 14, 1998
D. February 20,1995
Answer: D

53.Republic act. 7875 is also known as the :


a. National organ donation act of 1995
b. National health insurance act of 1995
c. Magna carta for health workers
d. None of the above
Answer: B

54.Republic Act 8423


A. created the PITAHC
B. provided for the Traditional and Alternative Health Development Fund
C. provided for the intensive and continuous scientific study on the herbal plants
D. all of the above
Answer: D

55.Consumer act of the Philippines or RA 7394 involves all of the following, except:
A. Protection against hazards to health and safety.
B. Provision of information and education to facilitate sound choice and the proper exercise of rights by the consumer.
C. Involvement of consumer representatives in the formulation of social and economic policies.
D. None of the above
Answer: D

56.The tool of family assessment known as SCREEM evaluates the following aspects of family life
A. Social
B. Cultural
C. Religious
D. All of the above
Answer: D

57.True about a family genogram


A. Graphically represents at least 3 generations
B. Includes even the deceased family members
C. Completely depicts family functioning
D. A and B
Answer: D

58. Family assessment instruments include


A. Family genogram
B. Family APGAR
C. Clinical biography and life events
D. All of the above
Answer: D

59.APGAR family assessment is interpreted by means of


A. Scoring
B. Comparing with a standard table
C. Using a scale of wellness
D. Consultation with a family psychologist
Answer: A

60.The APGAR family assessment measures


A. The newborn’s color, reflex, and cry
B. Satisfaction of members in the relationships within the family
C. The income of the family
D. The level of communication between husband and wife
Answer: B

61.True about family assessment


Should be done for all conscious patients and omitted for comatose patients
All the tools of assessment must be used
An interview with one member of the family is adequate
None of the above
Answer: D

62.Normal family function includes


A. Provide support to each other
B. Create rules that govern the conduct of family
C. Adapting to changes in the environment
D. All of the above
Answer: D

63.The statement that a family is a system means that


A. members have inherent inter-dependence
B. parents take the lead role
C. changes affect each member
D. A and C
Answer: D

64.The family as a unit of care is very special because


A. there is lifelong involvement
B. there are shared genetic and developmental attributes
C. they are confined to one household
D. A and B
Answer: D

65.Members of the nuclear family


A. parents
B. dependent children
C. spouses of the dependent children
D. A and B
Answer: D

66.Data on the family background is not needed when


A. Patient is comatose
B. Patient is caucasian
C. Patient is anti-social
D. None of the above
Answer: D
Basic areas of family function
A. Biologic
B. Economic
C. Educational
D. All of the above
Answer: D

68.It is the responsibility of the attending physician to explain the following


A. etiology of illness
B. pathophysiology of the illness
C. trajectory of outcome of the illness
D. All of the above
Answer: D

69.The illness trajectory begins with


A. the confirmatory laboratory results
B. the onset of symptoms
C. the onset of therapeutic efforts
D. recovery phase
Answer: B

70.Critical issues in choosing a therapeutic plan


A. financial cost
B. psychological state and preparedness of the patient and family
C. lifestyle and cultural characteristics of a family
D. All of the above
Answer: D

71.True about impact of acute illness in the family


A. emotions are high, and can lead to anger
B. family has no time for psychological adjustment
C. family members become sick in the course of the crisis
D. A and B
Answer: D

72.True about the illness trajectory


A. it is the pathological process of coping with illness
B. knowledge of the trajectory allows the physician to predict and anticipate family response to illness
C. it is the normal course of the psychosocial aspect of disease
D. B and C
Answer: D

The following statements are related to the impact of illness


A. Severe illness in parents place children of a family at great psychosocial difficulty
B. Illness sets in motion the processes that are disruptive of family life
C. Illness that is prolonged results in changes in family structure, roles and functions
D. All of the above
Answer: D

74.The five stages of family illness trajectory are the following except
A. Recovery phase
B. Onset of Illness to diagnosis
C. Termination phase
D. Major therapeutic efforts
Answer: C

Investigating the “illness” means


A. exploring the meaning of illness to patient and family
B. obtaining clinical history
C. obtaining laboratory test results
D. All of the above
Answer: A

The illness belief model may be influenced by


A. scientific medicine
B. religious beliefs
C. popular account
D. All of the above
Answer: D

During the impact phase, the physician’s responsibility include


A. Making clinical judgment about the amount of information the patient can absorb
B. Giving support and continuity of care
C. Clarify etiology of illness to address any feeling of guilt among family members
D. All of the above
Answer: D

78. Philhealth is
B. is a government collecting agency
C. is a government agency mandated by law to implement the NHIP
D. All of the above
Answer: C

79. Universal coverage means that PhilHealth aims to give medical coverage
A. to all Filipino citizens
B. to all Filipino citizens and foreign nationals
C. to natural born filipinos only
D. all of the above
Answer: A

80. Participation of health care providers in the NHIP


A. is compulsory
B. is a privilege
C. both A & B
D. neither A nor B
Answer: B

81. Surgical procedures covered by philhealth


A.laparoscopic procedures
B.caesarian section
C.excision biopsy
D.All of the above
Answer: D

82. Current outpatient benefits include


A. chemotherapy, hemodialysis, minor surgical procedures
B. chemotherapy, laboratory services,radiotherapy
C. hemodialysis, dental extraction, cataract extraction
D. all of the above
Answer: A

83. This law refers to the compulsory health insurance of the government
A. Medicare
B. Republic Act 7872
C. National Health Insurance Act of 1995
D. None of the above
Answer: C

84. NHIP ensures that health services are


A. affordable,acceptable,available,accessible
B. affordable,adjustable,admirable,negotiable
C. affordable and world class
D. none of the above
Answer: A

85 PhilHealth sets standards, guidelines and procedures prior to allowing doctors and hospitals
to become part of the NHIP. This process is called
A. evaluation
B. accreditation
C. recognition
D. acceleration
Answer: B

86. PhilHealth gives medical coverage to dependents of members. Dependents are the member’s
A. parents over 60, children below 21
B. parents over 65, children below 21
C. parents over 60, children below 18
D. parents over 65, children of any age
Answer: A

87. Health care provider in the definition of NHIP can be any of the following
A. doctor, under-board nurse, midwife
B. hospital, CBHO, HMO
C. doctor, hospital, faith healer
D. surgeon, anesthesiologist, medical intern
Answer: B

88. Non-paying philhealth member refers to


A. a member who has reached age 60
B. retiree who has paid 120 months of continuous contributions
C. delinquent members
D. A&B
Answer: D

89. Surgical procedures NOT covered by philhealth


A. laparoscopic procedures
B. caesarian section
C. silicone implantation, breast
D. excision biopsy
Answer: C

90. In the current EPI, a fully immunized child has


A. 1 BCG, 3 DPT, 3 OPV,1 MMR, 3 Hep B vaccine before 12 months of age
B. 2 BCG, 1 Measles vaccine, 3 DPT, 3 OPV before 12 months of age
C. 1 BCG, 3 DPT, 3 OPV, 1 Measles before 1 year of age
D. 1 BCG, 3 DPT, 3 OPV, 1 Measles, 3 Hep B vaccine before 1 year of age
Answer: D

91. The following are the target population of the EPI


A. all poor children at high risk areas
B. children 1-5 years old in measles high risk areas
C. all children without vaccination
D. infants 0 – 12 mos old, born to un-immunized mother
Answer: B

92. EPI was successful because it utilized different implementation strategies which include
A. monitoring, surveillance, health education
B. fee collection, social marketing
C. social mobilization, routine immunization day
D. A&C
Answer: D

93. Outbreak response means that


A. giving polio vaccine to children who have not completed their OPV once a case of polio is reported
B. giving OPV to all children under 5 regardless of immunization status once a case of polio is reported
C. giving measles immunization to contacts of a child with measles regardless of immunization status
D.giving all the EPI vaccinations to a child once a case of polio is reported in the area
Answer: B

94. In all levels of the government organization, there are four distinct personnel in the EPI, they are:
A. immunization officer, health educator
B. cold chain manager, disease surveillance officer
C. marketing officer, purchasing officer
D. A&B
Answer: D

95. It is given to women of child-bearing age


A. Tetanus immunoglobulin
B. Tetanus toxoid
C. Measles vaccine
D. BCG booster
Answer: B

96. When the child has the following condition, immunization is absolutely contraindicated
A. HIV infection
B. Adverse reaction to the previous immunization
C. A&B
D. None of the above
Answer: C

97.The goal(s) of the EPI is/are


A. Government revenue
B. Healthier workforce
C. Disease prevention
D. B&C
Answer: C
98. You may postpone immunization when the following conditions are present
A. when the child is hospitalized
B. when the child is febrile
C. when the child is taking antibiotics
D. A&B
Answer: D

99. You may proceed with immunization even when


A. the child has low grade fever
B. the child is preterm
C. then child is malnourished
D. All of the above
Answer: D

100. The following are examples of active immunization


A. BCG & OPV
B. BCG & DPT
C. BCG & Hepa B
D. All of the above
Answer: D

Recommended MPL: 60.5 (+/-) SD 0.1-1.0

University of the East


RAMON MAGSAYSAY MEMORIAL MEDICAL CENTER
College of Medicine
Department of Preventive and Community Medicine

1. The WHO definition of health states that HEALTH is:


A. the state of complete absence of all known disease
B. the harmonious functioning of the human body
C. the state of complete physical, mental and social well-being of the individual
D. feeling of wholeness

2. The three components of ecologic triad that influence occurrence and distribution of diseases are:
A. disease agent, spectrum of disease, environment
B. disease agent, host factors, environmental factors
C. environmental factors, disease agent, spectrum of disease
D. host, disease agent, spectrum of disease

3. This term refers to the ability of the biological agent to invade and enter the host and multiply:
A. infectivity
B. pathogenicity
C. virulence
D. antigenicity

4. The stage in the natural history of disease where interaction between agent, man and environment takes place to determine the actual
occurrence of disease in man, is:
A. prepathogenesis
B. incubation
C. clinical horizon
D. pathogenesis

5. Entry and multiplication of an infectious agent in the body of man or animal before signs and symptoms set in is:
A. pre-pathogenesis stage
B. actual infection
C. incubation period
D. convalescence

6. Pathogenesis stage starts when:


A. disease manifests itself with non-specific signs and symptoms
B. disease manifests itself with symptoms characteristic of the disease
C. once the infectious agent enters the host
D. any of the above

7. Nationwide educational campaign targeting high school students about the ill effects of smoking and alcohol, fall under what level of
prevention?
A. Primary level
B. Secondary level
C. Tertiary level
D. All of the above

8. The objectives of secondary level of prevention include/s:


A. to cure the disease
B. to minimize spread of the disease
C. to reduce serious consequences of the disease
D. All of the above

9. Mass screening for TB in a rural community using Sputum AFB Smear falls under what level of prevention?
A. Primordial
B. Primary
C. Secondary
D. Tertiary

10. Which of the following statements about tertiary level of prevention is correct?
A. it includes activities geared toward disability limitation
B. it prevents “social drift” among handicapped individuals
C. it includes psychological rehabilitation like crisis intervention
D. all of the above

11. A phenomenon where the proportion of immune individuals in the community has exceeded a significant level such that the infectious
organism is unable to maintain itself, leading to control of further transmission of the disease, is known as:
A. Mass immunization
B. Herd immunity
C. Infection control
D. All of the above

12. The following are characteristics of reservoir of infection, EXCEPT:


A. it is a source or setting where an infectious organism normally lives
B. it provides for the organism’s survival and multiplication
C. it permits transfer of infection to a susceptible host
D. None of the above

13. Examples of diseases that may be directly transmitted include all of the following, EXCEPT:
A. Hepatitis A
B. Rabies infection
C. Tuberculosis
D. Amoebiasis

14. The practice of close supervision of contracts for the purpose of prompt recognition of the infection without restriction of the their
movements, is called:
A. surveillance
B. isolation
C. segregation
D. quarantine

15. The practice of restriction the freedom of movement of a well person who has recently been exposed to a communicable disease, is
called:
A. surveillance
B. isolation
C. segregation
D. quarantine

16. Compared to formula milk, human milk has:


A. More proteins
B. More sugar
C. More vitamin A
D. All are correct

17. The most useful indicator of undernutrition:


A. Skinfold
B. Head circumference
C. Waist circumference
D. Weight for height

18. The following is an indirect method of nutritional assessment:


A. Weight for height
B. Percentage of low birth weight babies
C. Height for age
D. Measles mortality rate

19. The group most vulnerable to nutritional deficiencies:


A. Pregnant women
B. Working group
C. Adolescents
D. Non-lactating mothers

20. A clinical sign seen in kwashiorkor but not in marasmus:


A. Failure to thrive
B. Wasting
C. Edema
D. Attention deficit

21. The most sanitary source of water is:


A. Groundwater
B. Rainwater
C. Surface water
D. Brackish water

22. The primary disinfectant in water treatment:


A. Fluorine
B. Chlorine
C. Aluminum sulfate
D. Ferric sulfate

23. The process of water purification is as follows:


A. Coagulation, sedimentation, filtration, disinfection
B. Coagulation, filtration, disinfection, sedimentation
C. Sedimentation, coagulation, filtration, disinfection
D. Filtration, sedimentation, coagulation, disinfection

24. The material that falls to the bottom of the tank during water treatment is called:
A. Effluent
B. Precipitate
C. Sludge
D. Scum

25. Dissolved gases in water are removed by:


A. Filtration
B. Sedimentation
C. Chlorination
D. Aeration

26. The method of waste disposal wherein decomposition occurs through mild reactions mediated by bacteria in the ground over a long
period of time.
A. Incineration
B. Sanitary landfill
C. Dumping
D. Resource recovery

27. The preferred method of human waste management in urban areas:


A. Aqua privy
B. Septic tank
C. Chemical privy
D. Pit privy
28. Unsanitary waste disposal may be associated with the following disease:
A. Malaria
B. Tuberculosis
C. Trichinosis
D. Cholera

29. The qualities of a suitable latrine include:


A. Acceptable to people
B. Simple in construction and use
C. Hygienic and sanitary
D. All are correct

30. The preferred method of hospital waste disposal:


A. Incineration
B. Composting
C. Dumping
D. Landfill

31. Epidemiology is defined as:


A. a study of distribution and determinants of disease in human population
B. a study of the natural history of a disease
C. a study of disease prevention in human population
D. a study of etiologies of epidemic diseases in humans

32. The aim of Descriptive Epidemiology is/are:


A. to establish disease etiology
B. to provide general observations regarding a disease, as to person, place and time
C. to prove cause-and-effect relationship
D. all of the above

33. The component of epidemiology that involves testing of hypothesis of disease causation is known as:
A. Descriptive Epidemiology
B. Analytic Epidemiology
C. Inferential Epidemiology
D. Any of the above

34. Person characteristics that may be used to describe distribution of disease in a population include all, EXCEPT:
A. age
B. sex
C. ethnic group
D. None of the above

35. Which of the following patterns of occurrence of disease DOES NOT exemplify secular changes?
A. decline of stomach CA in the United States in the past 40 yrs
B. decline in deaths due to communicable diseases in the Philippines for the past 20 yrs
C. increased incidence of asthma attacks during cold months of December to February
D. increase in the morbidity rate of Diabetes in the Philippines in the last decade

36. A disease that occurs at an unusual or unexpected number is said to be


A. Epidemic
B. Endemic
C. Sporadic
D. Any of the above

37. The first step in outbreak investigation is:


A. establish the diagnosis
B. formulate the hypothesis
C. establish case definition
D. characterize afflicted individuals

38. The rate that measures the probability of a person dying in a year regardless of age, sex, race, etc is:
A. case fatality rate
B. neonatal mortality rate
C. crude death rate
D. specific death rate

39. The age specific death rate:


A. compares population with different age and sex composition
B. is generally higher among males than females
C. is higher in the urban areas than in rural
D. is higher in the extremes of ages

40. Used to describe the relative importance of different fatal disease in the population of different age, sex, occupation etc.
A. specific death rate
B. attack rate
C. cause of death rate
D. proportionate mortality rate

41. The number of death from a given cause in a specified time period divided by the total deaths in the same time peiod is
A. crude death rate
B. indirect death rate
C. case fatality rate
D. proportionate mortality ratio

42. The true mortality trend of a disease declines when there is


A. improved recognition of the disease
B. better prevention of the disease
C. better treatment of the disease
D. B and C

43. The following indices use the midyear population as the denominator, EXCEPT:
A. crude birth rate
B. measles incidence rate
C. crude death rate
D. proportionate mortality rate

44. Infant mortality of a community is determined by the number of deaths of infants less than one year old divided by:
A. the total number of births
B. the number of births
C. the number of babies born alive
D. the midyear population of infants less than 1 year

45. A measure of the risk of death during the first month (28 days) of life is:
A. perinatal mortality rate
B. neonatal mortality rate
C. infant mortality rate
D. fetal mortality rate

46. Post-neonatal mortality has been closely linked to:


A. environmental factor
B. maternal health prior to pregnancy
C. events during pregnancy
D. maternal health during pregnancy

47. A low infant mortality rate suggests:


A. good pre-post-natal systems
B. good infant and maternal nutrition
C. good environment sanitation
D. all of the above

48. The rate that measures the numbers of death from causes associated with delivery and diseases of the mother during pregnancy is:
A. infant mortality rate
B. neonatal mortality rate
C. maternal morbidity rate
D. maternal mortality rate

49. A proposed study on whether or not abortion should be legalized is not a good research problem because it does not satisfy this
criterion:
A. Intellectual curiosity
B. Researchability
C. Significance
D. Feasibility

50. A researcher comes across several studies which suggest that certain parameters in the chest X-ray are useful in assessing the
intravascular volume of critically ill patients. He decides to validate the study in the Philippines in the hope of identifying a possible
alternative to Swan Ganz catheterization, which is inaccessible and too expensive for most Filipinos. This method of identifying a potential
research problem is called:
A. Analysis of needs and practice
B. Serendipity
C. Intellectual curiosity
D. Organized and systematic determination of research needs

51. It is important to formulate a research hypothesis particularly when doing analytic studies because it helps the researcher in:
A. Identifying the most appropriate study design to use
B. Determining the best approach to how subjects are to be selected
C. Anticipating the appropriate statistical analysis to use
D. Estimating the sample size for the study

52. The schema for this study design essentially follows the same process as that of an experimental study:
A. Descriptive
B. Cross sectional
C. Case control
D. Cohort

53. The peculiar feature of a cross sectional study that distinguishes it from other study designs is:
A. Information on both the independent and dependent variables are collected simultaneously at the time of the survey
B. Information on the independent variable is collected first, after which the subjects are followed up later for assessment of the
dependent variable
C. Information on the dependent variable is obtained, after which assessment of the independent variable is made
D. Information on both the independent and dependent variables are obtained by doing either queries or actual observation

54. The effect of an inadequate sample size for a study is:


A. Subjects may not be a random representative of the base population
B. There may not be enough subjects that will develop the outcome, in order to detect a difference
C. The researcher will not be able to apply the appropriate statistical test for the kind of data available
D. The researcher might find an association between independent and dependent variables, even if there actually is none

55. The following method of data analysis is most relevant for the cohort study design:
A. Prevalence odds ratio
B. Prevalence rate
C. Odds ratio
D. Relative risk

56. This is a sampling method wherein every subject falling within a fixed sampling interval is obtained from a population N which is
arranged in some definite way:
A. Simple random sampling
B. Systematic sampling
C. Stratified sampling
D. Cluster sampling

57. In this sampling method, there is proportionate representation of the different classes or subgroups in the population:
A. Simple random sampling
B. Systematic sampling
C. Stratified sampling
D. Multi-stage sampling

58. Methods to increase the accuracy of observation and measurement include the following, EXCEPT:
A. Blinding
B. Training of observers
C. Formulation of operational definition
D. Sample size computation

59. Which method of collecting data is devoid of observer error and bias?
A. Interview
B. Self-administered questionnaire
C. Observation
D. Review of documents

60. This method of collecting data results to accumulation of “secondary data”:


A. self-administered questionnaire
B. observation
C. measurement
D. review of documents

61. The following statements are true regarding bias:


A. it may result both from biologic as well as measurement variations
B. it is a systematic deviation from a true underlying value
C. it may result from sampling error
D. All of the Above

62. Interpret the following: The 95% confidence interval of the relative risk is 2.0 to 4.0.
A. The standard deviation for the relative risk is ± 1.0.
B. There is a 95% probability that the relative risk is between 2.0 to 4.0.
C. The range of all possible risks for the development of the outcome is 2.0 to 4.0.
D. The variance for the relative risk is ± 1.0.

63. When a researcher wants to test for the association between two categorical variables, the appropriate statistic to use is:
A. Chi square
B. T test
C. Analysis of variance
D. Correlation coefficient

64. In situations where one extremely large or one extremely small observation occurs within a data set, the best measure of central
tendency to use is the:
A. Mean
B. Median
C. Mode
D. Quartile deviation

65. In a normal distribution, normal limits are delineated by:


A. The upper 5% of the observations
B. The lower 2.5% and the upper 2.5% of the observations
C. The lower 5% of the observations
D. The middle 95% of the observations

66. Statistical results of a study relating an independent and dependent variable show that p<0.05. What does this mean?
A. The study is significant enough for its results to be applied by readers in their practice
B. It is very likely that something other than the independent variable was responsible for the observed association
C. The probability that the observed difference is due to chance is very small
D. The independent variable affects the dependent variable to a clinically important extent

67. In studies dealing with performance of a diagnostic test, this term represents the proportion of diseased individuals with a (+) test result
A. Sensitivity
B. Positive predictive value
C. Likelihood ratio (+)
D. Validity

68. This term is synonymous with the True Negative Rate:


A. Specificity
B. Negative Predictive Value
C. Likelihood ratio (-)
D. Accuracy

69. BCG vaccine is given according to the Expanded Program on Immunization of the Philippines, at what age?
A. 3 months
B. 6 months
C. at birth
D. 9 months
70. The Expanded Program on Immunization is concerned with this particular age group:
A. Infants below 1 yr, school entrance, and pregnant women
B. Infants below 6 mos, pre-schoolers, and pregnant women
C. Neonates, pre-schoolers, and post-partum women
D. Infants of any age, school children, and pregnant women

71. In the EPI, one of the following schedules is not true


A. one dose of measles vaccine
B. two doses of BCG
C. three doses of primary DPT
D. three doses of primary OPV

72. A parameter that will help prioritize control measures on fertility to achieve impact on Family Planning Program:
A. general fertility rate
B. age-specific fertility rate
C. infant mortality rate
D. crude birth rate

73. The Family Planning Program consists of the following, EXCEPT:


A. promotion of knowledge on the use of contraceptive devices
B. helping childness couples to bear children
C. encouraging legal abortion
D. providing family planning services

74. During a diarrhea bout, breastfeeding of this infant should be


A. stopped and substitute with a lactose-free formula
B. temporarily discontinued
C. given more often
D. Limited

75. A program in the control of communicable diarrheal diseases by the Department of Health:
A. Hydrotherapy
B. oresol therapy
C. physiotherapy
D. use of intravenous fluids

76. The major activities undertaken in the maternal and child health program does NOT include
A. Immunize pregnant mothers against measles
B. Proper pneumonia prevention
C. Promote prenatal care
D. Monitor growth and health status of infants and children

77. The following are the principles of chemotherapy in Tuberculosis:


A. Drugs should be chosen to which the bacilli are likely to be susceptible.
B. Treatment must be started with one effective drug .
C. Bacteriostatic drugs are preferable to bactericidal drugs.
D. When treatment appears to be failing, add another single drug to
improve the situation.

78. The following is the correct definition of “treatment failure” in TB:


A. Patients who have been previously diagnosed as tuberculosis, who have had treatment with improvement or disappearance of
symptoms for one or more months, and now returned with clinical evidence of TB.
B. Patients who have failed to attend for 2 or more months in spite of every effort to trace them.
C. A new patient who is still sputum smear positive 5 months or more after starting standard treatment.
D. Patients who have moved to another area.

79. At the very least, sputum culture and sensitivity testing for Mycobacterium tuberculosis should be done whenever possible in the
following situations:
A. Smear (-) patients with a strong clinical possibility of PTB and suggestive chest X rays
B. Smear (+) or (-) patients of multi-drug resistant PTB
C. Smear (+) patients demonstrating the ‘rise fall’ phenomenon
D. All of the above

80. One of the following is NOT descriptive of a young population


A. population pyramid with a wide base
B. median age of 20-24 years
C. crude birth rate greater than 30/1000
D. dependency ratio of 1:1

81. The Philippine Health picture depicts the country’s poor state of health.
Which of the following is reflective of this?
A. increase in the life expectancy of the Filipinos
B. rising mortality rates from lifestyle related diseases
C. present male:female ratio
D. the rising inflation rate

82. The following are considered impediments to health:


A. poor living conditions
B. inability to make decisions about one’s health
C. inadequate education
D. all of the above

83. Based on the Alma Ata Declaration, the following were set by governments and the WHO:
A. freedom from all diseases
B. socially and economically productive life for the people
C. development of infrastructure for communities
D. free medicines for all

84. Elements of Primary Health Care include the following:


A. provision of essential drugs
B. proper nutrition for children and mothers
C. control of endemic diseases
D. all of the above

85. The objectives of Primary Health Care include the following:


A. acceptability, accessibility and affordability of health services
B. self-reliance in health care
C. environmental protection
D. all of the above

86. The expanded roles of a physician in Primary Health Care are:


A. social mobilizer
B. administrator of health programs
C. agents for change
D. all of the above

87. Important features of a Community-Based Health Program:


A. preset program objectives
B. managed solely by the people’s organization
C. people participation in all stages of development
D. availability of modern health equipments for community

88. Characteristics of Community-Oriented Primary Care programs:


A. authoritarian
B. paternalistic
C. democratic
D. liberating

89. Which of the following activities aims to gather information regarding the community’s health state and determinants?
A. community diagnosis
B. community integration
C. purok mobilization
D. education and training

90. The objectives of CBHP are set or formulated by the following:


A. representatives of the people
B. elected officials of the community
C. consultants and other experts
D. members of the clergy
91. One of the following is not a principal strategy utilized in a CBHP:
A. education and training of local human health resources
B. building people’s organization
C. establishment of relevant health services
D. building one hospital for every community

92. Distinct topic or group of tasks arranged over a definite period of time and place is known as:
A. task analysis
B. training curriculum
C. lesson plan
D. course programme

93. The following characterizes the curriculum for training CHW’s:


A. aims to prepare students for their work in PHC
B. this work/job will determine what will be taught rather than the teacher’s interest
C. it will develop the skills of the CHW so that the community they serve can achieve a high standard of health
D. all of the above

94. The basic principles of community organizing include the following, EXCEPT
A. mobilization of both internal and external resources
B. people have the capacity to change and to bring about change
C. the solution of problems commonly shared by the poorest sector entails organization, planning and action
D. start from the most complex problem because this needs the greatest considerations and the most resources

95. The following are true with regards to Community Organizing:


A. It is a tool used in achieving the PHC goals of self-reliance and self determination.
B. It is a method that can be used by community workers to generate participation in health services.
C. It is an approach used to prepare the community to eventually takeover the management of its health programs.
D. all of the above are true

96. The art and science of preventing diseases, promoting health and prolonging life through organized community effort, is:
A. Preventive Medicine
B. Community Medicine
C. Public Health
D. Social Medicine

97. The most cost-effective system of health care delivery is:


A. socialized health
B. cumpolsary health insurance
C. voluntary health insurance
D. free enterprise

98. Under the Local Government Code the District Health Office is under the:
A. Municipal/City Mayor
B. Provincial Governor
C. Regional Health Officer
D. Secretary of Health

99. The strongest aspect of production in the Philippine Health System is that of:
A. human health resources
B. biomedical technology
C. systems development
D. materials and products

100. The principal advantage of devolution of health services is:


A. decision making is closer to the people
B. graft and corruption is eliminated
C. primary health care is promoted
D. local government units can borrow money

SOURCES:
Beaglehole, Bonita, Kjellstrom; BASIC EPIDEMIOLOGY STUDENT’S TEXT, World Health Organization, July 1990
Clark, D., MacMahon, B: PREVENTIVE AND COMMUNITY MEDICINE, 2ND ed 1981
DOH Health Programs Manual
Gupta, M.; Mahajan, B: TEXTBOOK OF PREVENTIVE AND SOCIAL MEDICINE, 3rd ed 2003
Jekel, Elmore & Katz; EPIDEMIOLOGY BIOSTATISTICS AND PREVENTIVE MEDICINE, 1996
Kuzma, Bohnenblust; BASIC STATISTICS FOR THE HEALTH SCIENCES 4th ed., McGraw-Hill 2001
Last, J; Wallace, R; PUBLIC HEALTH AND PREVENTIVE MEDICINE, 13th ed, 1992
Philhealth Manual
Sanchez, Morelos, Baltazar, Peralta; RESEARCH METHODS IN HEALTH AND MEDICINE, Vol. 1 Planning Research (3rd ed.) Philippine
Council for Health Research and Development 1996
1. The term applied to an epidemic occurring within more than one country or territory:
A. Epidemic
B. Endemic
C. Pandemic
D. Sporadic

2. The causative agent for Avian flu is:


A. Influenza virus A
B. Influenza virus B
C. Coronavirus
D. Retrovirus

3. One of the following statements does not belong among the criteria for a SARS suspect:
A. Moderate to high-grade fever
B. Respiratory symptoms: cough, shortness of breath, or difficulty of breathing
C. History of contact with a person diagnosed with SARS, during the past 4 months
D. Travel to a country with known cases of SARS

4. The trees planted around the landfill help improve the quality of environmental air by making use of the emitted:
A. Methane
B. Carbon dioxide
C. Carbon monoxide
D. Oxygen

5. Methane is the product of the decomposition of which waste products?


A. Gaseous waste
B. Solid waste
C. Metal waste
D. Chemical waste

6. A Sanitary landfill may receive the following wastes, except:


A. Untreated hazardous wastes
B. Cardboard
C. Grass cuttings
D. Paper

7. Without the plastic lining at the bottom of the landfill, the leachate could:
A. Produce combustible gases
B. Seep through the soil and contaminate the water supply
C. Emit foul odor
D. Make the landfill sink deeper

8. The Zero Waste Management program is primarily intended to :


A. Extend the life of the sanitary landfill
B. Recycle metal and plastic material
C. Give extra income to people
D. Recycle paper into cardboard

9. The ideal way to dispose of hospital waste is by :


A. Burying in a landfill
B. Incineration
C. Grinding
D. Recycling

10. Passive immunity may be acquired by the following means, EXCEPT:


A. Intramuscular injection of immunoglobulin
B. Transfer of maternal antibodies across the placental barrier
C. Ingestion of colostum by the baby
D. Injection with viral antigen

11. Oral administration of vaccine is an effective route for which of the diseases?
A. Tetanus
B. Tuberculosis
C. Typhoid fever
D. Hemophilus influenza

Numbers 12 to 14 refer to Case 1:


Case 1 The patients who gave birth at the barangay health center were handled by the midwife. Some developed complications.
Prenatal No Prenatal
Age Group Number Number with Attack Number Number with Attack
complication Rate complication Rate
15 – 25 yrs. 33 15 45% 37 20 54%
26 – 36 yrs. 27 6 22% 31 11 35%
37 – 47 yrs 15 2 13% 12 11 92%

12. Case 1 : The age group/s with the highest total attack rate of complication is / are:
A. 15 – 25 yrs. C. 37 – 47 yrs.
B. 26 – 36 yrs. D. All of the above

13. Case 1 : The number of patients in the 15 – 25 yrs old group who had prenatal check-
up is :
A. 15 C. 45
B. 33 D. 48

14. Case 1 : The total number of patients without prenatal check-up who developed
complication is:
A. 20 C. 37
B. 32 D. 42

15. Which of the following is not among the top ten leading causes of morbidity in the Philippines?
A. Respiratory diseases
B. Diarrhea
C. Influenza
D. AIDS

16. A health care system that has the greatest impact on the health of a society, while making the best use of its resources, fulfills which of
the following values of social accountability?
A. Relevance
B. Quality
C. Cost-effectiveness
D. Equity

17. Availability of high-quality health care, to all sectors of society, especially the underprivileged, is the goal of which social accountability
value?
A. Relevance
B. Quality
C. Cost-effectiveness
D. Equity

18. The WHO’s Philippine counterpart in making health policies to answer local needs is the:
A. Association of Philippine Medical Colleges
B. Department of Health
C. City Health Office
D. Philippine Medical Association

19. The City health officer sent the sanitary inspector with medicine to treat the people of the town infested with scabies. The action taken is
a form of which level of prevention?
A. Primary prevention
B. Secondary prevention
C. Tertiary prevention
D. Risk assessment

20. Teaching uninfected households about sanitation and hygiene is what level of prevention?
A. Primary prevention
B. Secondary prevention
C. Tertiary prevention
D. Risk assessment

21. A student who wants to get health information of a given community may obtain it from the following, except:
A. Local government hospital
B. City health office
C. Provincial health office
D. Department of health

22. Epidemiology differs from clinical medicine in these regards, EXCEPT:


A. Epidemiologists study groups of people, not individuals
B. Epidemiologists study well people, in addition to sick people
C. Epidemiologists try to find out the crucial difference between those stricken and those spared
D. Epidemiologists decide optimal patient management

23. In the investigation of an epidemic of food poisoning at a banquet, high attack rates were found for people who ate roast beef as well as
those who ate mushroom sauce. Table 1 shows combinations of the two foods that were then considered.
Table 1 Attack Rates for Food Combinations

Ate Mushroom Sauce Did Not Eat Mushroom Sauce

Number Attack Number Attack


Number III Rate (%) Number III Rate (%)

Ate roast beef 150 105 70 72 2 3

Did not eat roast beef 42 33 78 26 0 0

Thus, the infective item is most likely to be


A. Mushroom sauce
B. Roast beef
C. The combination of roast beef with mushroom sauce
D. Cooking utensil (not the meat, nor the mushroom sauce)

24. What measures the proportion of the population dying every year or the number of deaths in the community per 1,000 populations.
A. Case fatality rate
B. Cause-specific mortality rate
C. Crude mortality rate
D. Proportionate mortality

25. Designed to measure the rate at which people without a disease develop the disease during a specific period of time, that is, the number of new
cases of a disease in a population over a period of time.
A. Fertility rates
B. Incidence rates
C. Mortality rates
D. Prevalence rates

26. Table 3 shows the sex distribution in three large series of cases of a disease.

Table 3 Sex Distribution in Three Series of Cases of a Disease


Series Male Cases Female Cases
1 200 100
2 250 50
3 450 150
Total 900 300

The incidence rate of this disease by sex was


A. Twice as great in males as in females
B. Three times greater in males than in females
C. Five times greater in make than in females
D. Cannot be computed from the date given

27. Which of the following measure the number of people in a population who have the disease at a given point in time.
A. Fertility rates
B. Incidence rates
C. Mortality rates
D. Prevalence rates

28. Epidemic refers to


A. A disease that has a low rate of occurrence but that is constantly present in a community or region
B. An attack rate in excess of 10 per 1,000 population
C. The occurrence of illnesses of similar nature clearly in excess of the normal expectation for that population at that time
D. The annual case rate per 100,000 population

29. Case fatality rate for a given disease refers to


A. The crude mortality rate per 100,000 population
B. Cause-specific mortality rate due to the disease
C. The percentage of deaths among cases of the disease
D. The proportion of deaths due to the disease among all deaths from all causes

30. The two major measures of disease frequency are:


A. Sensitivity and specificity
B. Positive and negative predictive values
C. Incidence and prevalence
D. Relative and Absolute risks

31. The ratio of the incidence of the group with the factor to the incidence of the group without the factor
A. Absolute risk
B. Attributable risk
C. Relative risk
D. Predictive value

32. Measures the amount of the incidence that can be attributed to one particular factor.
A. Absolute risk
B. Attributable risk
C. Relative risk
D. Predictive value

33. Sometimes called as retrospective study because of its direction of inquiry.


A. Case-Control Study
B. Cross-Sectional Study
C. Cohort Study
D. Experiment

34. Because the events of interest transpire after the study is begun, this study is sometimes called prospective study.
A. Case-Control Study
B. Cross-Sectional Study
C. Cohort Study
D. Experiment

35. Because it focuses on a point in time, it is sometimes also called prevalence study.
A. Case-Control Study
B. Cross-Sectional Study
C. Cohort Study
D. Experiment

36. A group of people who share a common experience within a defined time period.
A. Bias
B. Cohort
C. Placebo
D. Stratum

37. Which of the following is not a characteristic of a case-control study?


A. It is relatively inexpensive
B. Relative risk may be estimated from the results
C. Incidence rates may be computed
D. One selects controls without the disease

38. The strongest weapon in the scientific armamentarium to test a hypothesis.


A. Case-Control Study
B. Cross-Sectional Study
C. Cohort Study
D. Experiment

39. A well-known phenomenon in medicine is that patients given only inert substances will often show subsequent clinical improvement
when compared with similar patients not so “treated”
A. Block randomization
B. Placebo effect
C. Stratification
D. Surveillance

40. Defined as the detection of the occurrence of health-related events or exposures in a target population.
A. Predictive Value
B. Sensitivity
C. Specificity
D. Surveillance

41. The __________ of a test is defined as the percentage of persons with the disease of interest who have positive test results.
A. Negative Predictive Value
B. Positive Predictive Value
C. Sensitivity
D. Specificity

42. The __________ provides a way to look at risk in case-control studies.


A. Absolute Risk Reduction
B. Odds Ratio
C. Relative Risk
D. Relative Risk Reduction

43. The “four o’clock habit” of the Dengue Prevention campaign is about:
A. Immunization
B. Environmental sanitation
C. Personal hygiene
D. Anti-viral treatment

44. When educating the patient about Vitamin A, you will not include which of the following?
A. Yellow squash and carrots are rich food sources
B. Prevents night blindness
C. It is an anti-oxidant
D. Minimum daily requirement is 400 to 600 gms

45. The pathology in Vitamin A deficiency is:


A. Loss of muscle mass
B. Skin hyperpigmentation
C. Xerophthalmia
D. Exophthalmus

46. Inclusion of Iodized salt in the daily diet prevents:


A. Mental deficiency in children
B. Thin, brittle bones
C. Loose, non-elastic skin
D. Attention deficit hyperactivity disorder

47. Long-term psychiatric monitoring of a former drug user, who has been rehabilitated is a form of:
A. Primary prevention
B. Secondary prevention
C. Tertiary prevention
D. Quartiary prevention

48. The incubation period of AIDS is from the time:


A. Patient is infected up to the time AIDS–related symptoms manifest
B. The patient starts having large lymph nodes to the time severe lung infection manifests
C. Patient manifests mild symptoms until he/she dies
D. Patient starts feeling weak until pneumonia develops

49. The SARS virus is transmitted via :


A. Inhalation of infected droplets
B. Drinking contaminated water
C. Eating cooked infected chicken
D. Contact with infected blood

50. The characteristic clinical presentation of a patient with tuberculosis is a combination of which of the following:
A. Fever throughout the day, nosebleeding (epistaxis), loss of appetite
B. Cough of 2 weeks, evening fever, weight loss, loss of appetite
C. Fever of one week, abdominal enlargement, loss of appetite
D. Weight loss, large neck lymph nodes, anemia

51. Malfunctioning liver, distended abdomen and jaundice are symptoms consistent with:
A. Chronic alcoholism
B. AIDS
C. Protein deficiency
D. Carbohydrate deficiency

52. When taking the history of a woman whom you suspect to have STD, you would like to assess the possibility of exposure (risk
assessment) by asking about:
A. Number of children
B. When the symptoms started
C. Number of sexual partners
D. Health of parents

53. The incidence of tuberculosis is correctly stated as (using an arbitrary number):


A. 100,000 new cases per year
B. 100,000 tuberculous persons
C. 100,000 cases of newly diagnosed, plus undergoing treatment and healed TB cases
D. 100,000 cases confined in hospital

54. Prevalence of tuberculosis is correctly stated as (using an arbitrary number):


A. 2 billion TB cases in the world
B. 2 billion new TB cases in the world per year
C. 2 billion combined treated and healed TB cases per month
D. 2 billion TB cases, excluding the newly diagnosed cases

55. The capacity of a test to become negative in the absence of the disease is:
A. Specificity
B. Sensitivity
C. Predictive value of a negative test
D. Probability

56. The capacity of the test to become positive in the presence of the disease is its:
A. Specifity
B. Sensitivity
C. Predictive value
D. Probability

Numbers 57. to 60 refer to Case 3. Factory employees were screened for tuberculosis using the PPD tuberculin test and Chest x-ray.
Chest x-ray (+) Chest x-ray (-) Totals
Test (PPD) + A B A+B
Test (PPD) - C D C+D
Totals A+C B+D A+ B + C + D

57. Case 3: The group of subjects/employees with the highest probability of being
tuberculous :
A. A + C
B. A + B
C. A
D. B

58. Case 3: Total number of subjects/employees who are probably tuberculous:


A. A + C
B. B + D
C. A + B
D. C + D

59. Case 3: Predictive value of a positive chest xray:


A. A / A + B
B. B / A + B
C. A / A + C
D. B / B + D
60. Case 3: Predictive value of a negative chest xray:
A. A / A + B
B. B / A + B
C. A / A + C
D. B / B + D

61. A reduction in infant mortality may be interpreted as:


A. Increased life expectancy of infants
B. Increased life expectancy of infants and mothers
C. Decreased number of pregnancies
D. Effective family planning programs

62. Morbidity data would include the following, EXCEPT:


A. Accidents in the workplace
B. Deaths from accidents
C. Absences from work
D. Work-related illnesses

63. The best way to reduce the prevalence of heart diseases is to :


A. Train more cardiologists
B. Opening more coronary care units at the barangay level
C. Providing affordable medicines for cardiac diseases
D. Develop intensive health education programs to inform the communities about prevention

64. DOTS is Department of Health program for:


A. TB diagnosis
B. TB treatment
C. TB prevention
D. A and B only

65. Direct Observation Treatment , Short-course for tuberculosis is focused on which


level of prevention?
A. Primary
B. Secondary
C. Tertiary
D. A and B only

66. The Alma Ata Declaration is about which of the following?


A. Protection of women and children from violence
B. Health as a universal human right
C. Proper garbage disposal
D. Reduction of infant and maternal mortality

67. The science and art responsible for the maintenance and improvement of the health of the population, with police power to impose
sanctions on anyone that may harm the health of the public is:
A. Preventive medicine
B. Public health
C. General practice
D. Social medicine

68. If a clinician practicing in CDH suspects that a deep well is the source of typhoid fever of three of his patients, the person with the
power to have the water source shut down is:
A. Attending physician himself
B. Sanitary inspector
C. Cebu Doctors Hospital administrator
D. City health officer

69. The opposite of Equality between husband and wife is Paternalism, which means:
A. Dominance of men through their control of the family, commerce and society
B. Men are the superior members of the family through their active participation in household chores and upbringing of children
C. Men control external affairs, like business, while women control the home
D. Inheritance of wealth is from father to son only

70. The practice of financially-independent adult sons and daughters, who take time to go home during holidays demonstrates 3 of the
following characteristics, except:
A. Shared attributes
B. Belongingness
C. Lifelong membership
D. Child-centeredness

71. When a child kisses the hand of elders as a form of greeting, it is said that the parents have taught him/her well, fulfilling the following
family function, EXCEPT:
A. Biological
B. Socialization
C. Educational
D. Cultural

72. When a child is quarrelsome in school, does not mix well with his classmates, and does not answer when greeted by classmates, the
parents must have performed poorly in their _______________ function.:
A. Biological
B. Socialization
C. Educational
D. Cultural

Numbers 73 to 75 refer to Case 4: Ms. Ai-Ai has 3 children, aged 8 yrs. old, 5 yrs. 1 ½ yrs., by three different men , but she has remained
single. They live with Ai-ai’s aunt who has 2 grown up children, aged 29 and 30 years old. Ai-ai says that her children and her aunt and
cousins are the only family for her. What type family does Ai-ai have?

73. Case 4: What type of family does Ai-ai have?


A. Single parent, Nuclear family
B. Single parent, Extended family
C. Blended family
D. Polygamous family

74. Case 4: At what stage of family development is Ai-ai’s family?


A. Family with newborn child
B. Family with school-age children
C. Family with adolescent children
D. Launching family

75. Case 4: At what stage of development is the Auntie’s family?


A. Family with school-age children
B. Family with adolescent children
C. Launching family
D. Family in later years (Empty nest)

76. At which stage of family development do the parents begin to feel they are no longer
needed by their children?
A. Family with a new born child
B. Family with a child of school-age
C. Family with an adolescent child
D. Launching family

77. When the husband prevents the wife from going out with friends and her own parents, does not allow her to use the phone, and he
shouts and smashes her things when she disobeys him on such matters; Which of the following forms of controlling does not apply?
A. Isolation
B. Physical punishment
C. Intimidation
D. Emotional abuse

78. About 60 years ago, only women with higher IQ and grades, with extraordinary talent than the male competitors, were accepted in
medical schools. This is an example of :
A. Male-female power disparity
B. Socialization and learned behavior
C. Medicalization of the problem
D. Trivialization

79. The physician who is not aware of the issues on gender sensitivity, shows concern only for the cuts and bruises, and does not address
the emotional trauma and the risk to the patient’s life; such an attitude is called:
A. Male-female power disparity
B. Socialization and learned behavior
C. Medicalization of the problem
D. Trivialization

80. The woman who allows herself to be verbally abused by her husband must be educated about husband and wife relationship that is
based on:
A. Equality and Responsible parenting
B. Equality and Respect
C. Equality and Honesty
D. Equality and Economic partnership

81. The SARS virus may be acquired by:


A. Droplet infection
B. Airborne transmission
C. Sexual contact
D. Fecal-oral transmission

82. The carcinogenic substance in tobacco smoke is:


A. Carbon monoxide
B. Cyanide
C. Nicotine
D. Tar

83. Which of the following is a primary prevention activity for drug addiction?
A. Imprisonment of drug pushers
B. Responsible parenting seminar
C. Submission of a drug addict for rehabilitation
D. Drugs test for applicants of driver’s licence

84. Which of the following diseases cannot be prevented by hygienic methods of handling food and drinks?
A. Hepatitis A
B. Hepatitis B
C. Amebiasis
D. Typhoid fever

85. Which of the following conditions cannot be classified as a case of malnutrition?


A. Obesity
B. Kwashiorkor
C. Marasmus
D. Diabetes mellitus

86. Which of the following is/are solely the responsibility/ies of the public health practitioner?
A. Prompt diagnosis and treatment
B. Report the case to proper authorities
C. Trace and identify other sexual contacts of the index case
D. A and B only

87. The most common disease brought home by a child is:


A. Intestinal parasitism
B. Dengue fever
C. Respiratory tract infection
D. Measles

88. The appropriate term to describe Bino’s situation, who continues to smoke 2 packs of cigarettes daily inspite of the chronic cough is:
A. Tolerance to tobacco
B. Tobacco dependence
C. Addiction to tobacco
D. Misuse of tobacco

89. Bino has progressively increased the dose of diazepam, to put him to sleep, from 10 mg to 30 mg daily over 2 years, which is due to:
A. Tolerance
B. Resistance
C. Withdrawal
D. Misuse

90. Nurses and physicians must protect themselves from acquiring HIV infection by:
A. Getting vaccinated
B. Wearing a mask
C. Not touching the patient
D. Following precautionary measures in handling used hypodermic needles

91. The most powerful risk factor predisposing persons to atherosclerotic peripheral occlusive disease is:
A. Alcohol abuse
B. Cigarette smoking
C. Old age
D. Male gender

92. Being a good role-model for the children is about:


A A. Economic partnership
B. Responsible parenting
C. Honesty and accountability
D. Trust and support

93. Negotiation and fairness means:


A. Supporting her goals in life
B. Economic partnership
C. Earning trust and confidence through honesty
D. Seeking mutually satisfying resolution to conflict

94. Talking in loud, angry voices, when a couple is trying to find a solution to conflicting opinions is the opposite of :
A. Shared responsibility
B. Responsible parenting
C. Non-threatening behavior
D. Honesty and accountability

95. Which of the following functions is fulfilled when the children stay healthy because of completed immunizations?
A. Biological
B. Educational
C. Socialization
D. Cultural

96. Teaching the child to steal is a distortion of which function?


A. Biological
B. Educational
C. Socialization
D. Cultural

97. The type of abuse when the spouse does not speak to his partner, without any explanation, but only to express anger or dissatisfaction.
A. Physical
B. Sexual
C. Psychological
D. Social

98. The type of abuse, when the wife is not allowed to meet friends and family members; allowed to go out of the house.
A. Economic
B. Psychological
C. Social
D. Verbal

99. Blaming the rape victim for the assault, because she wore make-up and a mini-skirt (victim-blaming) is a promotive factor in the
causation of violence, which is known as:
A. Sex-role sterotype
B. Medicalization
C. Socialization
D. Trivialization

100. When the wife refuses to have sex with her husband and he forces her, he can be charged in court for:
A. Physical injuries
B. Marital rape
C. Battering
D. Frustrated homicide

DAVAO MEDICAL SCHOOL FOUNDATION


College of Medicine
PREVENTIVE MEDICINE

Multiple Choice: Encircle the correct answer.

1. The main thrust in the control of diarrhea cases by the Department of Health is:
A. Environmental Sanitation
B. Oral Rehydration Therapy with ORESOL
C. Public education
D. Immunization

2. Which of the following child survival strategies complement Oral Rehydration by decreasing the incidence and severity of diarrheal
infection?
A. Growth Monitoring
B. Immunization
C. Breastfeeding
D. Mother’s education on child care

3. The probability that a positive result is truly indicative of the disease or condition is:
A. None of these
B. Sensitivity
C. Specificity
D. positive

4. Public health services must give emphasis on:


A. Prevention, promotion and maintenance of health
B. Treatment of diseases
C. Rehabilitation of the differently abled individuals
D. Provision of emergency and first aid services

5. In public health, the most frequent problem is scarcity of resources. This can be best met by:
A. Transfer of funds
B. Compete for national funds
C. Allocate resources according to needs
D. Increase human resources training

6. In primary prevention the emphasis of health education should be on:


A. Recognition of health habits and customs
B. Case finding
C. Rehabilitation
D. Early diagnosis

7. The level of prevention directed towards preventing progression of the disease in community is:
A. Primary prevention
B. Secondary prevention
C. Tertiary prevention
D. Primordial prevention

8. The best route to block to prevent disease transmission is:


A. Portal of exit
B. Portal of entry
C. Migration in the body host
D. Vector control

9. Control measure/s directed against the vector:


A. Biological control
B. Mechanical control
C. Chemical control
D. All of the above

10. Some of the advantages of cohort:


A. Establish timing and directionality of events
B. Ideal to study rare exposure
C. Ability to evaluate several clinical outcomes
D. Ease in identifying comparison subjects

11. The DOH has ordered closure/cessation of all commercial blood banking facilities. Blood requirements of patients can be had by:
A. Blood substitutes
B. Voluntary blood donation
C. Component blood production
D. Hospital blood banks

12. In public health, the rate that is used measure illness is known as:
A. Morbidity rate
B. Natality rate
C. Mortality rate
D. Infectivity

13. The rate that reflects socio-economic condition of the community is:
A. Fertility rate
B. Infant mortality
C. Maternal mortality rate
D. A and B

14. Effective preventive medicine means stopping the morbid process when the:
A. Disease is progressing
B. Signs and symptoms are manifest
C. Patient is convalescing
D. Disease has not yet set in

15. The primary level of prevention includes:


A. Early case finding
B. Immunization
C. Disability limitation
D. Rehabilitation

16. The first level of preventions is most applicable:


A. Before host and agent interaction occur
B. During the initial interaction between the agent, the host and environment
C. During the convalescent stage of the disease
D. Al of the above

17. Which is the correct statement about Maternal Mortality


A. Higher among the very old and the very young
B. Higher among the primis than secundis
C. Higher among the secundis than the multipara
D. Lower in rural areas

18. Settling tank is a type of:


A. Preliminary treatment of water
B. Primary treatment of water
C. Tertiary treatment of water
D. Secondary treatment of water

19. Water is contaminated with fecal material when the test shows the presence of:
A. Planktons
B. Algae
C. Virus
D. Eschirichia coli

20. It is the most important single test to determine safety of water:


A. Bacteriological exam
B. Physical exam
C. Chemical e exam
D. Biological exam

21. The correct sequence in the process of water purification:


A. Coagulation, sedimentation, filtration distribution
B. Coagulation, sedimentation, filtration, disinfection
C. Sedimentation, coagulation, filtration distribution
D. Filtration, sedimentation, flocculation, disinfection

22. The principal carcinogens related to smoking:


A. Hydrocarbons
B. Nicotine
C. Carbon dioxide
D. Tar

23. Proportion of clinical cases resulting in severe clinical manifestations:


A. Pathogenecity
B. Virulence
C. Immunogenecity
D. Infectivity

24. Ability to produce clinically apparent illness:


A. Pathogenecity
B. Virulence
C. Immunogenecity
D. Infectivity

25. The most useful indicator of long-term nutritional deficiency:


A. Height for age
B. Middle-upper arm circumference
C. Weight for height
D. Weight for age

26. The most useful index for energy nutrient malnutrition in field Dudies relies on:
A. Questionnaire on dietary recall
B. Clinical signs of abnormal bed function
C. Anthropometrics measurement
D. Biochemical test

27. Laws and regulations on food sanitation are implemented to protect the public against fraud and deceit. Some of the restraints are the
following EXCEPT:
A. Misbranding
B. Sale of damaged food
C. Food adulteration
D. Adequate food storage

28. The Code of Sanitation of the Philippines is referred to as:


A. PD 1519
B. RA 3573
C. PD 965
D. PD 856

29. The distance of the burial ground from the source of water should be:
A. 100 meters
B. 75 meters
C. 30 feet
D. 10 feet

30. The period between the receipt of infection by a host and maximal communicability:
A. Herd immunity
B. Generation time
C. Lead time bias
D. Latency

31. STI has increased in an institute for the last 4 weeks. As public health officer, you should advice:
A. Better sanitation
B. Vaccination
C. Surveillance
D. Chemoprophylaxis

32. A 24 year old male presents with a 4-day history of burning sensation on urination. He also developed profuse creamy urethral
discharge. What would most likely be seen on the gram stain?
A. Numerous WBCs with intracellular gram-negative diplococci
B. Numerous WBCs with intracellular gram-positive diplococci
C. Numerous WBCs
D. Numerous WBCs with no bacteria

33. Resistance of a group to invasion and spread of an infectious agent, based on the immunity of a high proportion of individual members
of the group:
A. Protective factors
B. Active immunization
C. Passive Immunization
D. Herd immunity

34. In view of the cardiovascular disease, the principal strategy for their control should be:
A. Health education
B. Environmental control
C. Early diagnosis and treatment
D. Intensive therapy of cases

35. The minimum distance of a pit privy from a well in sandy soil is:
A. 10 meters
B. 15 meters
C. 20 meters
D. 25 meters

36. An essential health services that is available, accessible, affordable and culturally acceptable by the community:
A. Social Care
B. Preventive medicine
C. Medical care
D. Primary health care

37. The essential elements of primary health care:


A. Health education
B. Maternal and child birth
C. Control communicable diseases
D. All of these

38. Separation for the period of communicability


A. Quarantine
B. Isolation
C. Segregation
D. Personal surveillance

39. Which of the following has the greatest impact on community health?
A. Improvement of the standard of living
B. Provision of broad preventive medical measures
C. Provision of specific preventive measures
D. Provision of hospital care

40. The following activities and program supportive of CARI:


A. Nutrition
B. Anti-smoking and anti-pollution
C. Measles immunization
D. All of the above

41. Maternal and Child Health is concern with the health and well being of:
A. Child bearing women
B. Children
C. Potential parents
D. All of the above

42. Limitation of freedom of movement of such well persons


A. Quarantine
B. Isolation
C. Segregation
D. Personal surveillance

43. Requisites for successful parasitism include the following, EXCEPT:


A. Agent
B. Susceptible host
C. Proper climate
D. Satisfactory reservoir

44. The following are forms of quarantine EXCEPT:


A. Segregation
B. Modified quarantine
C. Segregation
D. No exceptions

45. The ultimate goal of malaria control eradication is:


A. Complete eradication of the plasmodium parasite
B. Total eradication of the mosquito vector
C. Treatment of all known cases
D. Vigilance against re-introduction

46. Immune globulin is given as preventive measure for many infectious diseases when:
A. Symptoms of the disease first appear
B. Exposure to a frank case has occurred
C. Recovery is taking place
D. Routine immunization is being instituted as well in baby clinics

47. In the Expanded Program of Immunization, BCG is given to:


A. Whenever the health of the child permits
B. Grade I children
C. Newborns
D. children 3-14 months of age

48. The recommended WHO schedule for measles vaccination is at weeks:


A. 6 weeks
B. 10 weeks
C. 3 months
D. 9 months

49. The most important initial management of rabies is:


A. Use of anti-biotic
B. Use of human diploid-cell
C. Irrigate saliva-containing bite wound
D. Catch and kill the dog

50. A 30 year old housemaid complains of progressive abdominal swelling, vague abdominal pain, low fever and weight loss. On PE the
abdomen is distended with a positive shifting dullness, aspiration reveals xanthochromic fluid with a gm/m PROTEIN (blood protein is 5).
This suggests:
A. Liver malignancy
B. TB peritonitis
C. Liver cirrhosis
D. Schistosomiasis

51. The following are different characteristics of hepatitis B from hepatitis A. EXCEPT:
A. Longer incubation period
B. Route of entry is mainly parental
C. Onset is more gradual
D. Occurs more in the younger age group

52. The planning of health education activity starts with:


A. Social preparation
B. Hypothesis formulation
C. Needs analysis
D. Budget setting

53. The primary objective of health education is to:


A. Transfer technology to lay personnel
B. Promote health in general
C. Impart knowledge
D. Improve health practices

54. The following is NOT included in the EPI Program of DOH:


A. Chickenpox
B. Measles
C. Diphtheria
D. Pertussis

55. The priority or target group in the anti-TB Program is:


A. Sputum negative but with suggestive of TB
B. Sputum negative cases but with possible x-ray findings
C. Sputum positive cases
D. Sputum negative but with clinical findings suggestive of TB

56. In TB symptomatic patient, how many sputum smears are recommended to establish diagnosis of TB?
A. Two
B. Four
C. Three
D. One

57. The DOH program “tutok gamutan” refers to:


A. Anti-tetanus vaccination
B. Polio vaccination
C. Micronutrient administration
D. Directly observed treatment strategy for TB
58. The following are forms to reduce susceptibility EXCEPT
A. Health promotion
B. Personal hygiene
C. Treatment
D. Prophylaxis

59. In measles, the period from exposure to appearance of rash is:


A. 3-6 days
B. 7-10 days
C. 12-14 days
D. 16-22 days

60. The vector of dengue fever is:


A. Aedes polcilus
B. Anpheles minimus
C. Aedes aegypti
D. Aedes seriation

61. Prevention of hepatitis can be achieved by:


A. Proper hygiene and sanitation
B. Needle and syringe sterilization
C. Screening of blood donors
D. Surveillance of all post-hepatitis
E. All of the above

62. Control has the following characteristics EXCEPT:


A. Cessation of transmission
B. Reduction of incidence
C. Prioritizes areas where transmission is high
D. Does not require total coverage of area

63. The first contact of the community to the health claim as defined by the PHC system is the:
A. Hospital personnel
B. Village health workers
C. Intermediate level health workers
D. Barangay captain

64. The following are true for volunteer community health workers, EXCEPT:
A. Establish linkage between government and non-government organizations
B. Are residents of the community
C. Provide only curative care
D. All of the above

65. The prime yardstick of child health level in the community is:
A. Infant and child mortality rate
B. Number of live births
C. Number of pre-school children
D. Number of health centers

66. Due to occurrence of flood and unsafe water supply, the most useful services that the local health unit can offer is/are:
A. Environmental sanitation
B. Communicable disease control
C. Health education
D. All of the above

67. In investigating an epidemic, cases should be categorized according to:


A. time, place and person
B. agent, host and environment
C. agent, host and date of onset
D. time, person and date of onset

68. At the beginning of the study, the participants are disease free individuals:
A. Cross-sectional
B. Cohort
C. Case-control using prevalent cases
D. Experimental study design

69. The strongest form of medical research study design is:


A. Clinical trial
B. Case report
C. Case-control study design
D. Cohort study design
70. One of the following is not an Analytical Study Design:
A. Prospective Cohort Study Design
B. Retrospective Cohort Study Design
C. Case Control Study Design
D. Population-Based Survey

71. Which of the following study designs may be prematurely terminated if interim analysis are found conclusive:
A. Case-control study design
B. Randomized Controlled Trial
C. Retrospective Cohort Study Design
D. Cross-Sectional Study Design

72. Study design in which exposure to a factor is known to all the study participants:
A. Cross-sectional
B. Cohort
C. Case-control using prevalent cases
D. Experimental study design

73. A study design used to provide prevalence of disease or other health outcomes in certain populations:
A. CASE report
B. Case series
C. Retrospective Cohort Study Design
D. Cross-sectional study design

74. One can be reasonably sure that the hypothesized cause preceded the occurrence of the disease and that disease status did not
differentially influence the selection of subjects by study factor level.
A. Cross-sectional
B. Cohort
C. Case-control using prevalent cases
D. Experimental study design

75. The preferred measure to determine causal association:


A. Ratio
B. Difference
C. Prevalence
D. None of the above

76. The measure that reflect the absolute number of cases attributable to the exposure:
A. Ratio
B. Difference
C. Prevalence
D. None of the above

77. The following decreases the prevalence of disease, except:


A. Out migration of susceptible cases
B. Improved immunization against the disease (if the disease is immunizable)
C. Treatment of the disease
D. None of the above

78. The following increases the prevalence of disease:


A. The disease has long duration but the incidence is low
B. The incidence is high but the duration of the disease is very short
C. The duration of the disease is long and the incidence is high
D. None of the above

79. This distribution is widely used because of its capacity to approximate other probabilities.
A. Binomial probability distribution
B. Poisson probability distribution
C. Normal probability distribution
D. None of the above

80. The group that we wish to study


A. Reference population
B. Target population
C. Study population
D. All of the above

81. Definition of mean


A. Summation of all values of sample points divided by the sample size
B. May have exactly the same sample points on both sides of the mean
C. A measure of central location
D. All of the above
E. None of the above
82. The difference between the highest observed value and the lowest observed value
A. Standard deviation
B. Mean
C. Range
D. Median
E. None of the above

83. The definition of variance


A. A measure of how the individual data points behave around the mean.
B. The square of a standard deviation
C. A measure of spread
D. Both b and c is correct
E. All of the above

84. This is an ordered display of each value in a data set together with its frequency, that is, the number of times that value occurs in the
data set.
A. Mean
B. Statistical table
C. Proportion
D. None of the above

85. What graphic methods that can be used if a variable is characterized by a numerical attribute, such as systolic blood pressure or birth
weight?
A. Horizontal bar diagram
B. Pie chart
C. Histogram
D. Vertical bar diagram

86. The following statement/s are true of bar diagram


A. For comparison of frequencies, whether absolute or relative
B. The magnitude for comparison is represented as bars whose lengths are proportionate to the values.
C. This is used of qualitative variables
D. All of the above

87. Which is true of the histogram


A. This is used for a qualitative variable
B. This is used for a quantitative discrete variable
C. This has the same use as the component bar
D. This is used for a quantitative continuous variable.
E. None of the above

88. The line diagram can be applied to this situation:


A. Qualitative variable
B. Time series
C. Discontinuous variable
D. All of the above
E. None of the above

89. Use/s of a scatterplot


A. Correlation data for two quantitative variable
B. Discontinuous variable
C. This is used for qualitative data
D. Both A and C are correct

90. What graphical presentation/s is/are best suited for distributions having a discrete basis of classification?
A. Pie diagram
B. Horizontal bar diagram
C. Vertical bar diagram
D. Both B and C are correct

91. What is a frequency polygon?


A. Used for qualitative data
B. Used for discontinuous data
C. Used for trend data
D. All of the above
E. None of the above

92. Which of the following is/are true of the principles of tabulation.


A. The tables should be simple. Two or three tables is better than one table with many variables
B. Each row and column should be labeled properly with specific units or measures for the data
C. The title should answer the questions what? Where? And when?
D. Totals should be shown
E. All of the above

93. The following descriptive statistics are very sensitive to extreme values.
A. Mean
B. Median
C. Standard deviation
D. Coefficient of variation
E. None of the above

94. The rationale of this measure is to ensure an equal number of sample points on both sides of the central location.
A. Mean
B. Median
C. Coefficient of variation
D. All of the above
E. None of the above

95. The principal strength of the sample is that it is insensitive to very large or very small values
A. Mean
B. Median
C. Mode
D. Coefficient of variation
E. None of the above

96. For distribution of infants according to the number of illness episodes experienced during the calendar year, this presentation is
appropriate.
A. Histogram
B. Frequency polygon
C. Vertical bar diagram
D. All of the above
E. None of the above

97. What presentation/s is/are appropriate for the distribution of children enrolled in the feeding program according to sex?
A. Frequency polygon
B. Pie chart
C. Vertical bar diagram
D. All of the above

98. The definition of variance


A. A measure of how the individual data points behave around the mean.
B. The square of a standard deviation
C. A measure of spread
D. All of the above

99. Large data sets are best describe using this/these:


A. Measure of spread
B. Measure of central tendency
C. Tabular presentation
D. Both a and b
E. All of the above

100. When one is interested to compare the variability of the height and the weight of the third year medical student, the best statistics to
use is/are:
A. Standard deviation
B. Variance
C. Coefficient of variation
D. All of the above
E. None of the above

***END***

Preventive Medicine Answer Key

1. C – MPL 90 11. C – MPL 90 21. A – MPL 75


2. C – MPL 90 12. C – MPL 75 22. D – MPL 75
3. C – MPL 75 13. B – MPL 75 23. A – MPL 75
4. B – MPL 75 14. D – MPL 75 24. C – MPL 75
5. B – MPL 90 15. D – MPL 75 25. B – MPL 75
6. A – MPL 75 16. C – MPL 50 26. D – MPL 75
7. B – MPL 50 17. D – MPL 50 27. D – MPL 75
8. A – MPL 75 18. B – MPL 50 28. C – MPL 75
9. B – MPL 75 19. B – MPL 75 29. C – MPL 75
10. D – MPL 75 20. A – MPL 90 30. C – MPL 75
31. C – MPL 50 57. C – MPL 75 83. B – MPL 90
32. B – MPL 50 58. A – MPL 75 84. B – MPL 90
33. A – MPL 75 59. C – MPL 50 85. D – MPL 90
34. C – MPL 75 60. D – MPL 50 86. C – MPL 75
35. B – MPL 75 61. A – MPL 75 87. C – MPL 90
36. B – MPL 50 62. B – MPL 75 88. C – MPL 75
37. C – MPL 75 63. D – MPL 75 89. A – MPL 75
38. D – MPL 75 64. D – MPL 90 90. D – MPL 90
39. B – MPL 50 65. B – MPL 90 91. B – MPL 75
40. D – MPL 50 66. B – MPL 75 92. B – MPL 90
41. C – MPL 75 67. B – MPL 75 93. D – MPL 90
42. B – MPL 50 68. D – MPL 90 94. C – MPL 90
43. B – MPL 90 69. A – MPL 50 95. A – MPL 90
44. D – MPL 90 70. D – MPL 75 96. B – MPL 90
45. C – MPL 90 71. A – MPL 75 97. C – MPL 75
46. A – MPL 90 72. B – MPL 75 98. C – MPL 75
47. C – MPL 75 73. B – MPL 75 99. D – MPL 75
48. A – MPL 75 74. B – MPL 75 100.B – MPL 90
49. A – MPL 75 75. C – MPL 75
50. B – MPL 75 76. D – MPL 90 References: Public Health and
51. A – MPL 90 77. B – MPL 75 ed.;
Preventive Medicine 14th
52. C – MPL 75 78. A – MPL 75
53. A – MPL 50 79. C – MPL 75 Robert Melbourne
54. B – MPL 50 80. B – MPL 75 Publisher: Application &
55. A – MPL 75 81. A – MPL 90
56. B – MPL 75 82. D – MPL 90 Lange

You might also like